Explore topic-wise InterviewSolutions in .

This section includes InterviewSolutions, each offering curated multiple-choice questions to sharpen your knowledge and support exam preparation. Choose a topic below to get started.

151.

Amory blaine inherite from his mother every trait, exept the stray inexpressible few, that made him worth while. His father, line an ineffectual, inarticulate man with a taste for byron and a habit of drowsing over the encyclopedia britannica, grew wealthy at thirty through the death of two elder brothers, successful chicago brokers, and in the first flush of feeling that the world was his, went to Bar harbor and met beatrice O ' hara. In consequence, stephen blaine handed down to posterity his height of just under six feet and his tendency to waver at crucial moments, these two abstractions appearing in his son amory. for many years he hovered in the background of his family's life, an unassertive figure with a face half-obliterated by lifeless, silky hair, continually occupied in taking care of his wife, continually harassed by the indea that he didn't and couldn't understander her. But Beartice blaine there was a woman early pictures taken on her fater's eastate at lake geneva, Wisconsin or in Rome at the sacrd heart convent _an educational extravagance that in her youth was only for the daughters of the exceptionally welthy showed the exquisite delicacy of her features, the consummate art and simplicity of her clothes. A brilliant education she had _ her youth passaed in renaissance glory, she was versed in the latest gossip of trhe older Roman families, known by name as a fabulously wealthy american girl to cardinal Vitori and Queen Margherita and more subtle celebrities that one must have had some culture even to have heard of. She learned in England to prefer whickey and soda to wine, and her small talk was broad ended in two senses during a winter in vienna. All in all beatrice O hara absorbed the sort of education that will be quite impossible ever again, a tutelage measured by the number of things and people one could be contemptuous of and charming about, a culture rich in all arts and traditions, barren of all ideas, in the last of those days when the great gardener clipped the inferior roses to produce one perfect bud. In her less important moments she returned to America, met stephen blaine and married him_ this almost entirely because she was a little bit weary, a little bit sad. Her only child was carried through a tiresome season and brought into the world on a spring day ininety _six. when amory was five he was already a delightful companion for her. He was an auburn-haired boy, with great, handsome eyes which he would grow up to in time, a facile imaginative maind and a taste for fancy dress. From his fourth to his tenth year he did the country with his mother in her fathers private car, from coronado, where f=his mother became so bord that she had nervous breakdown in a fashionable hotel down to Mexico city. where she took a mild, almost spidemic cosumption. this trouble pleased her, and later she made use of it as an intrinsic part of her atmosphere- especially after several astounding bracers. So while more or less fortunate little rich boys were defying governesses on the beach at newport, or being spanked or tutored or reqd to from " Do and Dare, " or " Frank on the Mississippi, " Amory was biting acquiescent bell -boys in the waldorf, outgrowing a natural repugnance to chamber music and symphonies, and deriving a highly special ized education froj his mother amory" " Yes, beatrice. " Such a quaint name for his mother, she encouraged it. dear, don't think of getting out of bed yet. I've always suspected that early rising in early life makes on nervous. Clothilde is having your breakfast brought up." " All right." I am feeling very old to day, amory, she would sigh, her face a rare coameo of pathos, her voice exquisityely modulated, her hands as facile as bernhardt's my nerves are on edge- on edge. We must leave this terrifying place to morrow and go searching for sunchine." Amory's penetrating gre3en eyes would look out through tangled hair at his m other even at this age he had no illusions about her. (Q) 9. As used in line 63, the word " did " most closely means

Answer»

MADE
traveled
caused
organized

Solution :In this CONTEXT, the narrator is referring to the fact that amory and his mother wandered the country for several years of his childhood, so traveled makes the most SENSE here. It is not accurate to say they made, caused, or organized the country in her father's PRIVATE car. the following sentences refer to a COUPLE of the locations they visited during their travels.
152.

Can you say how May 10 is 'an autumn day' in South Africa?

Answer»

SOLUTION :In South Africa, 10th of May was an autumn day, literally as well as symbolically. In autumn trees leave OLD and ROTTEN leaves and after that new leaves with fresh colours are born. Symbolically, the old and rotten system of apartheid was coming to an end. A new REPUBLIC BASED on equality of men, colours and races was taking birth on the 10th of May during the autumn.
153.

Change the following sentences into Indirect Speech : She said, ''Mahesh will be reading a book.''

Answer»

SOLUTION :She SAID that Mahesh WOULD be reading a BOOK.
154.

Carl struggled to regain his balance, but it was too late. He pitched forward and tumbled down the hill.

Answer»

inclined
stroked
fell
threw

Solution :(C ) Here, pitched means that Carl's BODY is falling froward as in (C ). Choices (A) and (D) USE other meanings of pitch (B) could either involve hitting SOMETHING or a condition of brain DAMAGE CAUSED by interruption of blood flow. Either way it is not a meaning of pitched.
155.

Belinda was braver than many bears put together. Ink and Blink were so brave that they could chase even lions down the stairs. Mustard, the dog, was as brave as a tiger in anger and rage. But Custard always cried like a coward to be put up in a nice safe cage. Why did Custard cry for a nice safe cage?

Answer»

Solution :Custard, the dragon was not conscious of his STRENGTH. Of all the pets of Belinda, Custard was really brave. But he ALWAYS underestimated himself. He used to cry like a COWARD and begged to be PUT into a nice safe cage.
156.

Belinda was braver than many bears put together. Ink and Blink were so brave that they could chase even lions down the stairs. Mustard, the dog, was as brave as a tiger in anger and rage. But Custard always cried like a coward to be put up in a nice safe cage. How were Belinda, Ink, Blink and Mustard brave?

Answer»

Solution :Belinda was braver than many bears put TOGETHER. Ink and Blink were brave too. They could chase even LIONS dawn the stairs. Mustard, the dog was LIKE a TIGER in RAGE.
157.

Change the following sentences into Indirect Speech : The Principal said to the peon, ''Let this boy go out.''

Answer»

Solution :The PRINCIPAL ORDERED the PEON to LET that BOY go out.
158.

After she had enough money saved, her next problem was how to slip out of the house without her mother's knowledge. But she managed this without too much difficulty. Every day after lunch her mother would nap from about one to four or so. Valli always used these hours for her 'excursions' as she stood looking from the doorway of her house or sometimes even ventured out into the village, today, these same hours could be used for her first excursion outside the village. How was the problem solved?

Answer»

Solution :The PROBLEM to slipping out of the house was SOLVED. Her mother would nap from about one to four or so. Valli COULD use the same hours for her first EXCURSION outside the village.
159.

After she had enough money saved, her next problem was how to slip out of the house without her mother's knowledge. But she managed this without too much difficulty. Every day after lunch her mother would nap from about one to four or so. Valli always used these hours for her 'excursions' as she stood looking from the doorway of her house or sometimes even ventured out into the village, today, these same hours could be used for her first excursion outside the village. What was the problem to she faced?

Answer»

SOLUTION :VALLEY had SAVED the MONEY. The only problem was how to slip out of the HOUSE. She wanted to slip out without her mother.s knowledge.
160.

1. Necessity is indeed the mother of invention. When areas in and around Leh began to experiencewater shortages, life didn't grind to a halt. Why? Because Chewang Norphel, a retired civilengineer in the Jammu and Kashmir government came up with the idea of artificial glaciers.2. Ladakh, a cold desert at an altitude of 3,000-3,500 metres above sea level, has a low averageannual rainfall rate of 50mm. Glaciers have always been the only source of water. Agriculture is completely dependent on glacier melt unlike the rest of river/monsoon-fed India. But over the years with increasing effects of climate change, rainfall and snowfall patterns have been changing, resulting in severe shortage and drought situations. Given the severe winterconditions, the window for farming is usually limited to one harvest season. 3. It is located between the natural glacier above and the village below. The one closer to thevillage and lowest in altitude melts first, providing water during April/May, the crucial sowing season. Further layers of ice above melt with increasing temperature thus ensuring continuous supply to the fields. Thus, farmers have been able to manage two crops instead of one. It costsabout 1,50,000 and above to create one. 4. Fondly called the “glacier man”, Mr. Norphel has designed over 15 artificial glaciers in andaround Leh since 1987. In recognition of his pioneering effort, he was conferred the PadmaShri by President Pranab Mukherjee, in 2015. 5. There are few basic steps followed in creating the artificial glacier. 6. River or stream water at higher altitude is diverted to a shaded area of the hill, facing north,where the winter sun is blocked by a ridge or a mountain range. At the start of winter/November, the diverted water is made to flow onto sloping hill face through distribution channels. Stone embankments are built at regular intervals which impede the flow of water, making shallow pools and freeze, forming a cascade of ice along the slope. Ice formation continues for 3-4 months resulting in a large accumulation of ice which is referred to as an “artificial glacier”. On the basis of your reading of the above passage answer the questions: The most appropriate sub-heading to para 6 of the passage is:

Answer»

River FORMATION
WATER for irrigation
Glaciers
Formation of an ARTIFICIAL Glacier

Solution : Formation of an Artificial Glacier.
161.

1. Necessity is indeed the mother of invention. When areas in and around Leh began to experiencewater shortages, life didn't grind to a halt. Why? Because Chewang Norphel, a retired civilengineer in the Jammu and Kashmir government came up with the idea of artificial glaciers.2. Ladakh, a cold desert at an altitude of 3,000-3,500 metres above sea level, has a low averageannual rainfall rate of 50mm. Glaciers have always been the only source of water. Agriculture is completely dependent on glacier melt unlike the rest of river/monsoon-fed India. But over the years with increasing effects of climate change, rainfall and snowfall patterns have been changing, resulting in severe shortage and drought situations. Given the severe winterconditions, the window for farming is usually limited to one harvest season. 3. It is located between the natural glacier above and the village below. The one closer to thevillage and lowest in altitude melts first, providing water during April/May, the crucial sowing season. Further layers of ice above melt with increasing temperature thus ensuring continuous supply to the fields. Thus, farmers have been able to manage two crops instead of one. It costsabout 1,50,000 and above to create one. 4. Fondly called the “glacier man”, Mr. Norphel has designed over 15 artificial glaciers in andaround Leh since 1987. In recognition of his pioneering effort, he was conferred the PadmaShri by President Pranab Mukherjee, in 2015. 5. There are few basic steps followed in creating the artificial glacier. 6. River or stream water at higher altitude is diverted to a shaded area of the hill, facing north,where the winter sun is blocked by a ridge or a mountain range. At the start of winter/November, the diverted water is made to flow onto sloping hill face through distribution channels. Stone embankments are built at regular intervals which impede the flow of water, making shallow pools and freeze, forming a cascade of ice along the slope. Ice formation continues for 3-4 months resulting in a large accumulation of ice which is referred to as an “artificial glacier”. On the basis of your reading of the above passage answer the questions: The ice formation continues for:

Answer»

2 MONTHS
3-4 months
5-6 months
7 months

Solution : 3-4 months.
162.

1. Necessity is indeed the mother of invention. When areas in and around Leh began to experiencewater shortages, life didn't grind to a halt. Why? Because Chewang Norphel, a retired civilengineer in the Jammu and Kashmir government came up with the idea of artificial glaciers.2. Ladakh, a cold desert at an altitude of 3,000-3,500 metres above sea level, has a low averageannual rainfall rate of 50mm. Glaciers have always been the only source of water. Agriculture is completely dependent on glacier melt unlike the rest of river/monsoon-fed India. But over the years with increasing effects of climate change, rainfall and snowfall patterns have been changing, resulting in severe shortage and drought situations. Given the severe winterconditions, the window for farming is usually limited to one harvest season. 3. It is located between the natural glacier above and the village below. The one closer to thevillage and lowest in altitude melts first, providing water during April/May, the crucial sowing season. Further layers of ice above melt with increasing temperature thus ensuring continuous supply to the fields. Thus, farmers have been able to manage two crops instead of one. It costsabout 1,50,000 and above to create one. 4. Fondly called the “glacier man”, Mr. Norphel has designed over 15 artificial glaciers in andaround Leh since 1987. In recognition of his pioneering effort, he was conferred the PadmaShri by President Pranab Mukherjee, in 2015. 5. There are few basic steps followed in creating the artificial glacier. 6. River or stream water at higher altitude is diverted to a shaded area of the hill, facing north,where the winter sun is blocked by a ridge or a mountain range. At the start of winter/November, the diverted water is made to flow onto sloping hill face through distribution channels. Stone embankments are built at regular intervals which impede the flow of water, making shallow pools and freeze, forming a cascade of ice along the slope. Ice formation continues for 3-4 months resulting in a large accumulation of ice which is referred to as an “artificial glacier”. On the basis of your reading of the above passage answer the questions:Glaciers provide water in the months of ______. It is crucial for irrigation.

Answer»

SOLUTION :April/May
163.

1. Necessity is indeed the mother of invention. When areas in and around Leh began to experiencewater shortages, life didn't grind to a halt. Why? Because Chewang Norphel, a retired civilengineer in the Jammu and Kashmir government came up with the idea of artificial glaciers.2. Ladakh, a cold desert at an altitude of 3,000-3,500 metres above sea level, has a low averageannual rainfall rate of 50mm. Glaciers have always been the only source of water. Agriculture is completely dependent on glacier melt unlike the rest of river/monsoon-fed India. But over the years with increasing effects of climate change, rainfall and snowfall patterns have been changing, resulting in severe shortage and drought situations. Given the severe winterconditions, the window for farming is usually limited to one harvest season. 3. It is located between the natural glacier above and the village below. The one closer to thevillage and lowest in altitude melts first, providing water during April/May, the crucial sowing season. Further layers of ice above melt with increasing temperature thus ensuring continuous supply to the fields. Thus, farmers have been able to manage two crops instead of one. It costsabout 1,50,000 and above to create one. 4. Fondly called the “glacier man”, Mr. Norphel has designed over 15 artificial glaciers in andaround Leh since 1987. In recognition of his pioneering effort, he was conferred the PadmaShri by President Pranab Mukherjee, in 2015. 5. There are few basic steps followed in creating the artificial glacier. 6. River or stream water at higher altitude is diverted to a shaded area of the hill, facing north,where the winter sun is blocked by a ridge or a mountain range. At the start of winter/November, the diverted water is made to flow onto sloping hill face through distribution channels. Stone embankments are built at regular intervals which impede the flow of water, making shallow pools and freeze, forming a cascade of ice along the slope. Ice formation continues for 3-4 months resulting in a large accumulation of ice which is referred to as an “artificial glacier”. On the basis of your reading of the above passage answer the questions: Ladakh is:

Answer»

a cold plain
a cold DESERT
a TEMPERATE desert
a HUMID desert

SOLUTION :a cold desert
164.

1. Necessity is indeed the mother of invention. When areas in and around Leh began to experiencewater shortages, life didn't grind to a halt. Why? Because Chewang Norphel, a retired civilengineer in the Jammu and Kashmir government came up with the idea of artificial glaciers.2. Ladakh, a cold desert at an altitude of 3,000-3,500 metres above sea level, has a low averageannual rainfall rate of 50mm. Glaciers have always been the only source of water. Agriculture is completely dependent on glacier melt unlike the rest of river/monsoon-fed India. But over the years with increasing effects of climate change, rainfall and snowfall patterns have been changing, resulting in severe shortage and drought situations. Given the severe winterconditions, the window for farming is usually limited to one harvest season. 3. It is located between the natural glacier above and the village below. The one closer to thevillage and lowest in altitude melts first, providing water during April/May, the crucial sowing season. Further layers of ice above melt with increasing temperature thus ensuring continuous supply to the fields. Thus, farmers have been able to manage two crops instead of one. It costsabout 1,50,000 and above to create one. 4. Fondly called the “glacier man”, Mr. Norphel has designed over 15 artificial glaciers in andaround Leh since 1987. In recognition of his pioneering effort, he was conferred the PadmaShri by President Pranab Mukherjee, in 2015. 5. There are few basic steps followed in creating the artificial glacier. 6. River or stream water at higher altitude is diverted to a shaded area of the hill, facing north,where the winter sun is blocked by a ridge or a mountain range. At the start of winter/November, the diverted water is made to flow onto sloping hill face through distribution channels. Stone embankments are built at regular intervals which impede the flow of water, making shallow pools and freeze, forming a cascade of ice along the slope. Ice formation continues for 3-4 months resulting in a large accumulation of ice which is referred to as an “artificial glacier”. On the basis of your reading of the above passage answer the questions: Chewang Norphel was a:

Answer»

RETIRED civil engineer
retired SCHOOL master
retired army officer
WORKING journalist

Solution :retired civil engineer
165.

1. Necessity is indeed the mother of invention. When areas in and around Leh began to experiencewater shortages, life didn't grind to a halt. Why? Because Chewang Norphel, a retired civilengineer in the Jammu and Kashmir government came up with the idea of artificial glaciers.2. Ladakh, a cold desert at an altitude of 3,000-3,500 metres above sea level, has a low averageannual rainfall rate of 50mm. Glaciers have always been the only source of water. Agriculture is completely dependent on glacier melt unlike the rest of river/monsoon-fed India. But over the years with increasing effects of climate change, rainfall and snowfall patterns have been changing, resulting in severe shortage and drought situations. Given the severe winterconditions, the window for farming is usually limited to one harvest season. 3. It is located between the natural glacier above and the village below. The one closer to thevillage and lowest in altitude melts first, providing water during April/May, the crucial sowing season. Further layers of ice above melt with increasing temperature thus ensuring continuous supply to the fields. Thus, farmers have been able to manage two crops instead of one. It costsabout 1,50,000 and above to create one. 4. Fondly called the “glacier man”, Mr. Norphel has designed over 15 artificial glaciers in andaround Leh since 1987. In recognition of his pioneering effort, he was conferred the PadmaShri by President Pranab Mukherjee, in 2015. 5. There are few basic steps followed in creating the artificial glacier. 6. River or stream water at higher altitude is diverted to a shaded area of the hill, facing north,where the winter sun is blocked by a ridge or a mountain range. At the start of winter/November, the diverted water is made to flow onto sloping hill face through distribution channels. Stone embankments are built at regular intervals which impede the flow of water, making shallow pools and freeze, forming a cascade of ice along the slope. Ice formation continues for 3-4 months resulting in a large accumulation of ice which is referred to as an “artificial glacier”. On the basis of your reading of the above passage answer the questions:Mr. Norphel has been called:

Answer»

the NOBLE man
the GREAT man
the mountain man
the GLACIER man

SOLUTION : the glacier man
166.

But in the hearts of all who lived in the solitary house in the middle of the valley, there was a single hope:help from God. Don't be so upset, even thoughb this seems like a total loss.Remember no one dies of hunger." "That's what they say: no one dies of hunger." What was the only hope left among those who were living in that solitary house in the valley?

Answer»

Solution :EVERYTHING was DESTROYED by the hail storm. The WIDESPREAD destruction left Lencho and his family on the VERGE of starvation. They had only one hope. It was some KIND of help from God.
167.

But in the hearts of all who lived in the solitary house in the middle of the valley, there was a single hope:help from God. Don't be so upset, even thoughb this seems like a total loss.Remember no one dies of hunger." "That's what they say: no one dies of hunger." How did they console themselves in that time of difficulty?

Answer»

Solution :No doubt, the HAIL had destroyed all the crops and cornfields. Lencho and his family had come on the verge of starvation. HOWEVER, they had a firm faith in God and in His mercy and help. They CONSOLED themselves that God WOULD not ALLOW them to die of hunger.
168.

But if it has to perish twice I think I know enough of hate. What does 'it' refer to? How will it perish twice?

Answer»

Solution :.It. here refers to the world. The POET says that PEOPLE hold different opinions about the end of the world. Some say "FIRE", which stands for desire., will DESTROY the world. Secondly, .ice., which stands for . HEATED . can also destroy the world.
169.

At birth, the child was very fair and pretty. But when she was two years old, she had an attack of small-pox. Only the eyes were saved, but the entire body was permanently disfigured by deep black pockmarks. Little Sulekha could not speak till she was five, and when at last she learnt to speak, she stammered. The other children often made fun of her and mimicked her. As a result, she talked very little.What did she remember?

Answer»

SOLUTION : She REMEMBERED her cow Laxmi. Only a few days ago, their old cow had been TURNED out and sold. BHOLI doubted if the same fate was AWAITING her.
170.

At birth, the child was very fair and pretty. But when she was two years old, she had an attack of small-pox. Only the eyes were saved, but the entire body was permanently disfigured by deep black pockmarks. Little Sulekha could not speak till she was five, and when at last she learnt to speak, she stammered. The other children often made fun of her and mimicked her. As a result, she talked very little.Why was Bholi frightened?

Answer»

Solution :Actually, Bholi.s FATHER was asked by the TEHSILDAR to set an example to the villagers by SENDING her daughters to SCHOOL. When Ram Lal caught Bholi by the hand to take her to school, she was FRIGHTENED. Actually, she had never stepped out of the house.
171.

Change the following sentences into Passive Voice. He hurt his leg in an accident.

Answer»

SOLUTION :His LEG was HURT in an ACCIDENT.
172.

A Natural SyntheticIn 1970, Norman Borloug was awarded the Nobel Prize and credited with saving over a billion people from starvation. In what is now called the Green Revolution, Borlaug led the research and development over a two-decade span beginning inthe 1940's to dramatically increases agricultural production worldwide. He introduced the synthetic farming methods already common in the United Stated and Britain to a global market,focusing particularly on the developing world and secceeded in hiking food production and saving lives. Borlauf's intiative calls for celebration.Yet,it is these same agricultural techniques-those associated with conventional farming-that have a bad over the last twenty-five years, causing the organic food market to a sour to a whopping $63 billion by 2012. These laborsaving,high-yeilding techniques bgan in the late 18th centuary and were perfected for nearly two centuries before worry spread that they seriously harmed the soil and allowed toxic chemicals to enter the food supply. In the 1940's while Borloug was busy feeding the world's impoverished, Albert and Gabrielle Howard-both accomplished botanists-were developing organic agriculture. Organic farming is the process by which crops are raised usnig only natural methods to maintain soil fertility and control pests.In the current food market, GMO's or genetically modified organisms, turn noses faster than saturated fats and soda pop. Instead,organic farmers rely on crop rotation, green manure and biological pest control, while excluding synthetic fertilizers,pesticides and growth hormones. Organic agriculture is said to promote sustainability, openness, health and safety and it standard are closely regulated by the International Federation of organic Agriculture MOvements.The IFOAM bases the foundation of organic farming on the minimal use of oo-farm inputs and on management practices that restore,preserve and improve ecological harmony. While this strategy sounds more "conventional" and effortless that what is now coined conventional farming, organic agriculture is actually quite scientific.Ecologically,organic farming is designed to promote and enhance biodiversity, so it must combine scientific knowledge and technologies to stimulate naturally occuring biological processes. For instance, organic farming uses pyrethrin, a natural pesticide found in the chrysanthemum flower, to deter pests and potassium bicarbonate to control diseases and suppress unruly weeds. Furthermore,where conventional farming focuses on mass production of each individual crop,organic farming encourages polyculture or multiple crops being raised in the same space . To replace nutrients, organic farming relies on the natural breakdown of organic matter by microorganisms like mycorrhiza, which forms a symbiotic relationship between fungi and plant roots. To relenish nitrogen, green manure is created by leaving uprooted crop parts to wither on a feild and is then used as a cover crop to fix nitrogen into the soil.The science doesn't stop with the crops. On farms with livestock, the feild of agroecology-which includes organic agriculture-attempts to provide animals with natural living conditions and feed. Just like in plants,organic farming rejects any growth hormones or genetic engineering in animals. The USDA has specific regulations in regard to organic livestock,demanding that the animals recieves only orgaic feed and are pastured rather than caged.Despite the popularity of organic foods, many argue that the concerns over conventional farming are a luxury of the rich. Organic farming yeild far less than conventional methods, uses more land and more labour, and is, therefore, more expensive.When prices rise and population falls, it is the poor that suffer. With the United Nations reporting 870 million people worldwide suffering from chronic malnutrition, organic farming faces a tough arguement against the capital-intensive, prolific conventional means. Science has a lot more work to do before organic agricultural methods can feed the world. Q.The information in the graph gives the most direct evidence in support of a claim made in

Answer»

lines 9-11 ("Yet…2012")
lines 19-21 ("Instead…hormones")
Lines 21-23 ("Organic…Movements")
lines 46-47 ("Organic…EXPENSIVE")

SOLUTION :(D) The graph shows crop prices, and in CASE, organic prices are higher than conventional prices for that sae crop. So, the graph supports lines 46-47, which state that organic farming is more expensive. (A) illustrate the RECENT growth of the organic MARKET, while (B) and (C ) give details of organic techniques.
173.

A Natural SyntheticIn 1970, Norman Borloug was awarded the Nobel Prize and credited with saving over a billion people from starvation. In what is now called the Green Revolution, Borlaug led the research and development over a two-decade span beginning inthe 1940's to dramatically increases agricultural production worldwide. He introduced the synthetic farming methods already common in the United Stated and Britain to a global market,focusing particularly on the developing world and secceeded in hiking food production and saving lives. Borlauf's intiative calls for celebration.Yet,it is these same agricultural techniques-those associated with conventional farming-that have a bad over the last twenty-five years, causing the organic food market to a sour to a whopping $63 billion by 2012. These laborsaving,high-yeilding techniques bgan in the late 18th centuary and were perfected for nearly two centuries before worry spread that they seriously harmed the soil and allowed toxic chemicals to enter the food supply. In the 1940's while Borloug was busy feeding the world's impoverished, Albert and Gabrielle Howard-both accomplished botanists-were developing organic agriculture. Organic farming is the process by which crops are raised usnig only natural methods to maintain soil fertility and control pests.In the current food market, GMO's or genetically modified organisms, turn noses faster than saturated fats and soda pop. Instead,organic farmers rely on crop rotation, green manure and biological pest control, while excluding synthetic fertilizers,pesticides and growth hormones. Organic agriculture is said to promote sustainability, openness, health and safety and it standard are closely regulated by the International Federation of organic Agriculture MOvements.The IFOAM bases the foundation of organic farming on the minimal use of oo-farm inputs and on management practices that restore,preserve and improve ecological harmony. While this strategy sounds more "conventional" and effortless that what is now coined conventional farming, organic agriculture is actually quite scientific.Ecologically,organic farming is designed to promote and enhance biodiversity, so it must combine scientific knowledge and technologies to stimulate naturally occuring biological processes. For instance, organic farming uses pyrethrin, a natural pesticide found in the chrysanthemum flower, to deter pests and potassium bicarbonate to control diseases and suppress unruly weeds. Furthermore,where conventional farming focuses on mass production of each individual crop,organic farming encourages polyculture or multiple crops being raised in the same space . To replace nutrients, organic farming relies on the natural breakdown of organic matter by microorganisms like mycorrhiza, which forms a symbiotic relationship between fungi and plant roots. To relenish nitrogen, green manure is created by leaving uprooted crop parts to wither on a feild and is then used as a cover crop to fix nitrogen into the soil.The science doesn't stop with the crops. On farms with livestock, the feild of agroecology-which includes organic agriculture-attempts to provide animals with natural living conditions and feed. Just like in plants,organic farming rejects any growth hormones or genetic engineering in animals. The USDA has specific regulations in regard to organic livestock,demanding that the animals recieves only orgaic feed and are pastured rather than caged.Despite the popularity of organic foods, many argue that the concerns over conventional farming are a luxury of the rich. Organic farming yeild far less than conventional methods, uses more land and more labour, and is, therefore, more expensive.When prices rise and population falls, it is the poor that suffer. With the United Nations reporting 870 million people worldwide suffering from chronic malnutrition, organic farming faces a tough arguement against the capital-intensive, prolific conventional means. Science has a lot more work to do before organic agricultural methods can feed the world. Q.Generalizing based on the graph, one could estimate that organic produces is approximately

Answer»

half as expensive as conventional.
one FOURTH as expensive as conventional.
four times as expensive as expensive as conventional.

Solution :(C ) Since organic is CLEARLY more expensive, we can rule out (A) and (B). Moreover, looking at the GRAPH, the organic numbers are approximately double that of the non-organic numbers for the same CROP. (D) is MUCH too extreme for the data provided.
174.

A Natural SyntheticIn 1970, Norman Borloug was awarded the Nobel Prize and credited with saving over a billion people from starvation. In what is now called the Green Revolution, Borlaug led the research and development over a two-decade span beginning inthe 1940's to dramatically increases agricultural production worldwide. He introduced the synthetic farming methods already common in the United Stated and Britain to a global market,focusing particularly on the developing world and secceeded in hiking food production and saving lives. Borlauf's intiative calls for celebration.Yet,it is these same agricultural techniques-those associated with conventional farming-that have a bad over the last twenty-five years, causing the organic food market to a sour to a whopping $63 billion by 2012. These laborsaving,high-yeilding techniques bgan in the late 18th centuary and were perfected for nearly two centuries before worry spread that they seriously harmed the soil and allowed toxic chemicals to enter the food supply. In the 1940's while Borloug was busy feeding the world's impoverished, Albert and Gabrielle Howard-both accomplished botanists-were developing organic agriculture. Organic farming is the process by which crops are raised usnig only natural methods to maintain soil fertility and control pests.In the current food market, GMO's or genetically modified organisms, turn noses faster than saturated fats and soda pop. Instead,organic farmers rely on crop rotation, green manure and biological pest control, while excluding synthetic fertilizers,pesticides and growth hormones. Organic agriculture is said to promote sustainability, openness, health and safety and it standard are closely regulated by the International Federation of organic Agriculture MOvements.The IFOAM bases the foundation of organic farming on the minimal use of oo-farm inputs and on management practices that restore,preserve and improve ecological harmony. While this strategy sounds more "conventional" and effortless that what is now coined conventional farming, organic agriculture is actually quite scientific.Ecologically,organic farming is designed to promote and enhance biodiversity, so it must combine scientific knowledge and technologies to stimulate naturally occuring biological processes. For instance, organic farming uses pyrethrin, a natural pesticide found in the chrysanthemum flower, to deter pests and potassium bicarbonate to control diseases and suppress unruly weeds. Furthermore,where conventional farming focuses on mass production of each individual crop,organic farming encourages polyculture or multiple crops being raised in the same space . To replace nutrients, organic farming relies on the natural breakdown of organic matter by microorganisms like mycorrhiza, which forms a symbiotic relationship between fungi and plant roots. To relenish nitrogen, green manure is created by leaving uprooted crop parts to wither on a feild and is then used as a cover crop to fix nitrogen into the soil.The science doesn't stop with the crops. On farms with livestock, the feild of agroecology-which includes organic agriculture-attempts to provide animals with natural living conditions and feed. Just like in plants,organic farming rejects any growth hormones or genetic engineering in animals. The USDA has specific regulations in regard to organic livestock,demanding that the animals recieves only orgaic feed and are pastured rather than caged.Despite the popularity of organic foods, many argue that the concerns over conventional farming are a luxury of the rich. Organic farming yeild far less than conventional methods, uses more land and more labour, and is, therefore, more expensive.When prices rise and population falls, it is the poor that suffer. With the United Nations reporting 870 million people worldwide suffering from chronic malnutrition, organic farming faces a tough arguement against the capital-intensive, prolific conventional means. Science has a lot more work to do before organic agricultural methods can feed the world. Q. As used in line 36, the word "symbiotic" most closely means

Answer»

enviornmentally significant
hierarchicallly predatory.
mutually beneficial.
agriculturally resilient.

Solution :(C ) "Symbiotic" can be defined as the close ASSOCIATION of two organism in a mutually beneficial relatioship. In line 36, we see that microorganisms from symbiotics relationships between fungus and PLANT roots to naturally break down orgaic matter and replace nutrients. Therefore,evven without knowing the definition,we can infer that this relationship is favourable to all. (B) provides an opposite meaning from this. (A), while true, is not descriptive of the advantages relationship. (D) is incorrect because there is no indication that these ORGANISMS have experienced a HARDSHIP that demands an agricultural resolation-they are simply helping out one anther.
175.

A Natural SyntheticIn 1970, Norman Borloug was awarded the Nobel Prize and credited with saving over a billion people from starvation. In what is now called the Green Revolution, Borlaug led the research and development over a two-decade span beginning inthe 1940's to dramatically increases agricultural production worldwide. He introduced the synthetic farming methods already common in the United Stated and Britain to a global market,focusing particularly on the developing world and secceeded in hiking food production and saving lives. Borlauf's intiative calls for celebration.Yet,it is these same agricultural techniques-those associated with conventional farming-that have a bad over the last twenty-five years, causing the organic food market to a sour to a whopping $63 billion by 2012. These laborsaving,high-yeilding techniques bgan in the late 18th centuary and were perfected for nearly two centuries before worry spread that they seriously harmed the soil and allowed toxic chemicals to enter the food supply. In the 1940's while Borloug was busy feeding the world's impoverished, Albert and Gabrielle Howard-both accomplished botanists-were developing organic agriculture. Organic farming is the process by which crops are raised usnig only natural methods to maintain soil fertility and control pests.In the current food market, GMO's or genetically modified organisms, turn noses faster than saturated fats and soda pop. Instead,organic farmers rely on crop rotation, green manure and biological pest control, while excluding synthetic fertilizers,pesticides and growth hormones. Organic agriculture is said to promote sustainability, openness, health and safety and it standard are closely regulated by the International Federation of organic Agriculture MOvements.The IFOAM bases the foundation of organic farming on the minimal use of oo-farm inputs and on management practices that restore,preserve and improve ecological harmony. While this strategy sounds more "conventional" and effortless that what is now coined conventional farming, organic agriculture is actually quite scientific.Ecologically,organic farming is designed to promote and enhance biodiversity, so it must combine scientific knowledge and technologies to stimulate naturally occuring biological processes. For instance, organic farming uses pyrethrin, a natural pesticide found in the chrysanthemum flower, to deter pests and potassium bicarbonate to control diseases and suppress unruly weeds. Furthermore,where conventional farming focuses on mass production of each individual crop,organic farming encourages polyculture or multiple crops being raised in the same space . To replace nutrients, organic farming relies on the natural breakdown of organic matter by microorganisms like mycorrhiza, which forms a symbiotic relationship between fungi and plant roots. To relenish nitrogen, green manure is created by leaving uprooted crop parts to wither on a feild and is then used as a cover crop to fix nitrogen into the soil.The science doesn't stop with the crops. On farms with livestock, the feild of agroecology-which includes organic agriculture-attempts to provide animals with natural living conditions and feed. Just like in plants,organic farming rejects any growth hormones or genetic engineering in animals. The USDA has specific regulations in regard to organic livestock,demanding that the animals recieves only orgaic feed and are pastured rather than caged.Despite the popularity of organic foods, many argue that the concerns over conventional farming are a luxury of the rich. Organic farming yeild far less than conventional methods, uses more land and more labour, and is, therefore, more expensive.When prices rise and population falls, it is the poor that suffer. With the United Nations reporting 870 million people worldwide suffering from chronic malnutrition, organic farming faces a tough arguement against the capital-intensive, prolific conventional means. Science has a lot more work to do before organic agricultural methods can feed the world. Q. Lines 39-41 serve to demonstrate that

Answer»

animals that have been raised naturally are more content.
providing animals with unnatural feed is DELETERIOUS to their health.
agroecology is the FIRST attend to unify science with animals husbandry.
organic techniques have ALSO been applied to raising animals.

Solution :(D) These lines go on to SHOW the science behinf organic livestock, so (D) captures their PURPOSE. Choices (A), (B) and (C ) ar all assuming.
176.

A Natural SyntheticIn 1970, Norman Borloug was awarded the Nobel Prize and credited with saving over a billion people from starvation. In what is now called the Green Revolution, Borlaug led the research and development over a two-decade span beginning inthe 1940's to dramatically increases agricultural production worldwide. He introduced the synthetic farming methods already common in the United Stated and Britain to a global market,focusing particularly on the developing world and secceeded in hiking food production and saving lives. Borlauf's intiative calls for celebration.Yet,it is these same agricultural techniques-those associated with conventional farming-that have a bad over the last twenty-five years, causing the organic food market to a sour to a whopping $63 billion by 2012. These laborsaving,high-yeilding techniques bgan in the late 18th centuary and were perfected for nearly two centuries before worry spread that they seriously harmed the soil and allowed toxic chemicals to enter the food supply. In the 1940's while Borloug was busy feeding the world's impoverished, Albert and Gabrielle Howard-both accomplished botanists-were developing organic agriculture. Organic farming is the process by which crops are raised usnig only natural methods to maintain soil fertility and control pests.In the current food market, GMO's or genetically modified organisms, turn noses faster than saturated fats and soda pop. Instead,organic farmers rely on crop rotation, green manure and biological pest control, while excluding synthetic fertilizers,pesticides and growth hormones. Organic agriculture is said to promote sustainability, openness, health and safety and it standard are closely regulated by the International Federation of organic Agriculture MOvements.The IFOAM bases the foundation of organic farming on the minimal use of oo-farm inputs and on management practices that restore,preserve and improve ecological harmony. While this strategy sounds more "conventional" and effortless that what is now coined conventional farming, organic agriculture is actually quite scientific.Ecologically,organic farming is designed to promote and enhance biodiversity, so it must combine scientific knowledge and technologies to stimulate naturally occuring biological processes. For instance, organic farming uses pyrethrin, a natural pesticide found in the chrysanthemum flower, to deter pests and potassium bicarbonate to control diseases and suppress unruly weeds. Furthermore,where conventional farming focuses on mass production of each individual crop,organic farming encourages polyculture or multiple crops being raised in the same space . To replace nutrients, organic farming relies on the natural breakdown of organic matter by microorganisms like mycorrhiza, which forms a symbiotic relationship between fungi and plant roots. To relenish nitrogen, green manure is created by leaving uprooted crop parts to wither on a feild and is then used as a cover crop to fix nitrogen into the soil.The science doesn't stop with the crops. On farms with livestock, the feild of agroecology-which includes organic agriculture-attempts to provide animals with natural living conditions and feed. Just like in plants,organic farming rejects any growth hormones or genetic engineering in animals. The USDA has specific regulations in regard to organic livestock,demanding that the animals recieves only orgaic feed and are pastured rather than caged.Despite the popularity of organic foods, many argue that the concerns over conventional farming are a luxury of the rich. Organic farming yeild far less than conventional methods, uses more land and more labour, and is, therefore, more expensive.When prices rise and population falls, it is the poor that suffer. With the United Nations reporting 870 million people worldwide suffering from chronic malnutrition, organic farming faces a tough arguement against the capital-intensive, prolific conventional means. Science has a lot more work to do before organic agricultural methods can feed the world. Q.Based on the graph, the difference in price between the organic of a crop and the conventional version of the crop is greatest with

Answer»

corn
wheat
soyabean
none of these, as It depends on the particular year considered.

Solution :(C ) The QUESTION is ASKING you to find the greatest deviation between a crop's organic and no-organic PRICE. Since, organic soyabean is much more EXPENSIVE than the other crop prices, we can see that this divergence is greatest.
177.

A Natural SyntheticIn 1970, Norman Borloug was awarded the Nobel Prize and credited with saving over a billion people from starvation. In what is now called the Green Revolution, Borlaug led the research and development over a two-decade span beginning inthe 1940's to dramatically increases agricultural production worldwide. He introduced the synthetic farming methods already common in the United Stated and Britain to a global market,focusing particularly on the developing world and secceeded in hiking food production and saving lives. Borlauf's intiative calls for celebration.Yet,it is these same agricultural techniques-those associated with conventional farming-that have a bad over the last twenty-five years, causing the organic food market to a sour to a whopping $63 billion by 2012. These laborsaving,high-yeilding techniques bgan in the late 18th centuary and were perfected for nearly two centuries before worry spread that they seriously harmed the soil and allowed toxic chemicals to enter the food supply. In the 1940's while Borloug was busy feeding the world's impoverished, Albert and Gabrielle Howard-both accomplished botanists-were developing organic agriculture. Organic farming is the process by which crops are raised usnig only natural methods to maintain soil fertility and control pests.In the current food market, GMO's or genetically modified organisms, turn noses faster than saturated fats and soda pop. Instead,organic farmers rely on crop rotation, green manure and biological pest control, while excluding synthetic fertilizers,pesticides and growth hormones. Organic agriculture is said to promote sustainability, openness, health and safety and it standard are closely regulated by the International Federation of organic Agriculture MOvements.The IFOAM bases the foundation of organic farming on the minimal use of oo-farm inputs and on management practices that restore,preserve and improve ecological harmony. While this strategy sounds more "conventional" and effortless that what is now coined conventional farming, organic agriculture is actually quite scientific.Ecologically,organic farming is designed to promote and enhance biodiversity, so it must combine scientific knowledge and technologies to stimulate naturally occuring biological processes. For instance, organic farming uses pyrethrin, a natural pesticide found in the chrysanthemum flower, to deter pests and potassium bicarbonate to control diseases and suppress unruly weeds. Furthermore,where conventional farming focuses on mass production of each individual crop,organic farming encourages polyculture or multiple crops being raised in the same space . To replace nutrients, organic farming relies on the natural breakdown of organic matter by microorganisms like mycorrhiza, which forms a symbiotic relationship between fungi and plant roots. To relenish nitrogen, green manure is created by leaving uprooted crop parts to wither on a feild and is then used as a cover crop to fix nitrogen into the soil.The science doesn't stop with the crops. On farms with livestock, the feild of agroecology-which includes organic agriculture-attempts to provide animals with natural living conditions and feed. Just like in plants,organic farming rejects any growth hormones or genetic engineering in animals. The USDA has specific regulations in regard to organic livestock,demanding that the animals recieves only orgaic feed and are pastured rather than caged.Despite the popularity of organic foods, many argue that the concerns over conventional farming are a luxury of the rich. Organic farming yeild far less than conventional methods, uses more land and more labour, and is, therefore, more expensive.When prices rise and population falls, it is the poor that suffer. With the United Nations reporting 870 million people worldwide suffering from chronic malnutrition, organic farming faces a tough arguement against the capital-intensive, prolific conventional means. Science has a lot more work to do before organic agricultural methods can feed the world. Q.The paragraph in lines 28-38 functions to counter the claim that organc agricluture

Answer»

lacks a scientific approach.
is relatively costly.
has nutritional merit.
causes widespread starvation.

Solution :(A) Lines 28-38 EXPLAIN the science BEHIND ORGANIC agriculture in response to the previous claim that it occurs naturally, so (A) is the BEST answer. (B) and (C ) are done in other parts of the passage. And (D) is untrue.
178.

A Natural SyntheticIn 1970, Norman Borloug was awarded the Nobel Prize and credited with saving over a billion people from starvation. In what is now called the Green Revolution, Borlaug led the research and development over a two-decade span beginning inthe 1940's to dramatically increases agricultural production worldwide. He introduced the synthetic farming methods already common in the United Stated and Britain to a global market,focusing particularly on the developing world and secceeded in hiking food production and saving lives. Borlauf's intiative calls for celebration.Yet,it is these same agricultural techniques-those associated with conventional farming-that have a bad over the last twenty-five years, causing the organic food market to a sour to a whopping $63 billion by 2012. These laborsaving,high-yeilding techniques bgan in the late 18th centuary and were perfected for nearly two centuries before worry spread that they seriously harmed the soil and allowed toxic chemicals to enter the food supply. In the 1940's while Borloug was busy feeding the world's impoverished, Albert and Gabrielle Howard-both accomplished botanists-were developing organic agriculture. Organic farming is the process by which crops are raised usnig only natural methods to maintain soil fertility and control pests.In the current food market, GMO's or genetically modified organisms, turn noses faster than saturated fats and soda pop. Instead,organic farmers rely on crop rotation, green manure and biological pest control, while excluding synthetic fertilizers,pesticides and growth hormones. Organic agriculture is said to promote sustainability, openness, health and safety and it standard are closely regulated by the International Federation of organic Agriculture MOvements.The IFOAM bases the foundation of organic farming on the minimal use of oo-farm inputs and on management practices that restore,preserve and improve ecological harmony. While this strategy sounds more "conventional" and effortless that what is now coined conventional farming, organic agriculture is actually quite scientific.Ecologically,organic farming is designed to promote and enhance biodiversity, so it must combine scientific knowledge and technologies to stimulate naturally occuring biological processes. For instance, organic farming uses pyrethrin, a natural pesticide found in the chrysanthemum flower, to deter pests and potassium bicarbonate to control diseases and suppress unruly weeds. Furthermore,where conventional farming focuses on mass production of each individual crop,organic farming encourages polyculture or multiple crops being raised in the same space . To replace nutrients, organic farming relies on the natural breakdown of organic matter by microorganisms like mycorrhiza, which forms a symbiotic relationship between fungi and plant roots. To relenish nitrogen, green manure is created by leaving uprooted crop parts to wither on a feild and is then used as a cover crop to fix nitrogen into the soil.The science doesn't stop with the crops. On farms with livestock, the feild of agroecology-which includes organic agriculture-attempts to provide animals with natural living conditions and feed. Just like in plants,organic farming rejects any growth hormones or genetic engineering in animals. The USDA has specific regulations in regard to organic livestock,demanding that the animals recieves only orgaic feed and are pastured rather than caged.Despite the popularity of organic foods, many argue that the concerns over conventional farming are a luxury of the rich. Organic farming yeild far less than conventional methods, uses more land and more labour, and is, therefore, more expensive.When prices rise and population falls, it is the poor that suffer. With the United Nations reporting 870 million people worldwide suffering from chronic malnutrition, organic farming faces a tough arguement against the capital-intensive, prolific conventional means. Science has a lot more work to do before organic agricultural methods can feed the world. Q. As used in line 5, the word "common" most closely means

Answer»

widespread
lowly
COMMUNAL
corparate

Solution :(A) Lines 5 refers to Borlaug's contribution to worldwide poverty and hunger, stating that he extended the FARMING methods "already common In the United States and Britain." Since this is reffering to their prevalence, widespread is the correct COICE. Lowly means of LOW importance. Communal and corprate would imply that the methods were literally shared members of a community.
179.

A Natural SyntheticIn 1970, Norman Borloug was awarded the Nobel Prize and credited with saving over a billion people from starvation. In what is now called the Green Revolution, Borlaug led the research and development over a two-decade span beginning inthe 1940's to dramatically increases agricultural production worldwide. He introduced the synthetic farming methods already common in the United Stated and Britain to a global market,focusing particularly on the developing world and secceeded in hiking food production and saving lives. Borlauf's intiative calls for celebration.Yet,it is these same agricultural techniques-those associated with conventional farming-that have a bad over the last twenty-five years, causing the organic food market to a sour to a whopping $63 billion by 2012. These laborsaving,high-yeilding techniques bgan in the late 18th centuary and were perfected for nearly two centuries before worry spread that they seriously harmed the soil and allowed toxic chemicals to enter the food supply. In the 1940's while Borloug was busy feeding the world's impoverished, Albert and Gabrielle Howard-both accomplished botanists-were developing organic agriculture. Organic farming is the process by which crops are raised usnig only natural methods to maintain soil fertility and control pests.In the current food market, GMO's or genetically modified organisms, turn noses faster than saturated fats and soda pop. Instead,organic farmers rely on crop rotation, green manure and biological pest control, while excluding synthetic fertilizers,pesticides and growth hormones. Organic agriculture is said to promote sustainability, openness, health and safety and it standard are closely regulated by the International Federation of organic Agriculture MOvements.The IFOAM bases the foundation of organic farming on the minimal use of oo-farm inputs and on management practices that restore,preserve and improve ecological harmony. While this strategy sounds more "conventional" and effortless that what is now coined conventional farming, organic agriculture is actually quite scientific.Ecologically,organic farming is designed to promote and enhance biodiversity, so it must combine scientific knowledge and technologies to stimulate naturally occuring biological processes. For instance, organic farming uses pyrethrin, a natural pesticide found in the chrysanthemum flower, to deter pests and potassium bicarbonate to control diseases and suppress unruly weeds. Furthermore,where conventional farming focuses on mass production of each individual crop,organic farming encourages polyculture or multiple crops being raised in the same space . To replace nutrients, organic farming relies on the natural breakdown of organic matter by microorganisms like mycorrhiza, which forms a symbiotic relationship between fungi and plant roots. To relenish nitrogen, green manure is created by leaving uprooted crop parts to wither on a feild and is then used as a cover crop to fix nitrogen into the soil.The science doesn't stop with the crops. On farms with livestock, the feild of agroecology-which includes organic agriculture-attempts to provide animals with natural living conditions and feed. Just like in plants,organic farming rejects any growth hormones or genetic engineering in animals. The USDA has specific regulations in regard to organic livestock,demanding that the animals recieves only orgaic feed and are pastured rather than caged.Despite the popularity of organic foods, many argue that the concerns over conventional farming are a luxury of the rich. Organic farming yeild far less than conventional methods, uses more land and more labour, and is, therefore, more expensive.When prices rise and population falls, it is the poor that suffer. With the United Nations reporting 870 million people worldwide suffering from chronic malnutrition, organic farming faces a tough arguement against the capital-intensive, prolific conventional means. Science has a lot more work to do before organic agricultural methods can feed the world. Q. An unstated assumption of the author is that

Answer»

organic agriculture helps PROMOTE diversity of crops.
agriculture is the way that most people will ACQUIRE food.
many people in the world have a lack of sustenance.
organic agriculture is RELATIVELY popular.

Solution :(B) Since (A),(C ) and (D) are all started within the PASSAGE, choice (B) is the correct answer.
180.

A Natural SyntheticIn 1970, Norman Borloug was awarded the Nobel Prize and credited with saving over a billion people from starvation. In what is now called the Green Revolution, Borlaug led the research and development over a two-decade span beginning inthe 1940's to dramatically increases agricultural production worldwide. He introduced the synthetic farming methods already common in the United Stated and Britain to a global market,focusing particularly on the developing world and secceeded in hiking food production and saving lives. Borlauf's intiative calls for celebration.Yet,it is these same agricultural techniques-those associated with conventional farming-that have a bad over the last twenty-five years, causing the organic food market to a sour to a whopping $63 billion by 2012. These laborsaving,high-yeilding techniques bgan in the late 18th centuary and were perfected for nearly two centuries before worry spread that they seriously harmed the soil and allowed toxic chemicals to enter the food supply. In the 1940's while Borloug was busy feeding the world's impoverished, Albert and Gabrielle Howard-both accomplished botanists-were developing organic agriculture. Organic farming is the process by which crops are raised usnig only natural methods to maintain soil fertility and control pests.In the current food market, GMO's or genetically modified organisms, turn noses faster than saturated fats and soda pop. Instead,organic farmers rely on crop rotation, green manure and biological pest control, while excluding synthetic fertilizers,pesticides and growth hormones. Organic agriculture is said to promote sustainability, openness, health and safety and it standard are closely regulated by the International Federation of organic Agriculture MOvements.The IFOAM bases the foundation of organic farming on the minimal use of oo-farm inputs and on management practices that restore,preserve and improve ecological harmony. While this strategy sounds more "conventional" and effortless that what is now coined conventional farming, organic agriculture is actually quite scientific.Ecologically,organic farming is designed to promote and enhance biodiversity, so it must combine scientific knowledge and technologies to stimulate naturally occuring biological processes. For instance, organic farming uses pyrethrin, a natural pesticide found in the chrysanthemum flower, to deter pests and potassium bicarbonate to control diseases and suppress unruly weeds. Furthermore,where conventional farming focuses on mass production of each individual crop,organic farming encourages polyculture or multiple crops being raised in the same space . To replace nutrients, organic farming relies on the natural breakdown of organic matter by microorganisms like mycorrhiza, which forms a symbiotic relationship between fungi and plant roots. To relenish nitrogen, green manure is created by leaving uprooted crop parts to wither on a feild and is then used as a cover crop to fix nitrogen into the soil.The science doesn't stop with the crops. On farms with livestock, the feild of agroecology-which includes organic agriculture-attempts to provide animals with natural living conditions and feed. Just like in plants,organic farming rejects any growth hormones or genetic engineering in animals. The USDA has specific regulations in regard to organic livestock,demanding that the animals recieves only orgaic feed and are pastured rather than caged.Despite the popularity of organic foods, many argue that the concerns over conventional farming are a luxury of the rich. Organic farming yeild far less than conventional methods, uses more land and more labour, and is, therefore, more expensive.When prices rise and population falls, it is the poor that suffer. With the United Nations reporting 870 million people worldwide suffering from chronic malnutrition, organic farming faces a tough arguement against the capital-intensive, prolific conventional means. Science has a lot more work to do before organic agricultural methods can feed the world. Q. Based on the passage, when compared to a non-organic farm feild, an organic farm field will most likely be more

Answer»

productive
diverse
mature
CENTRALIZED

Solution :(B) Accourding to the author, organic FARMING "encourages polyculture," so it PRODUCES more diversity. The passage states that conventional farming is more productive and centralized. There is no evidence for (C ) either way.
181.

A Natural SyntheticIn 1970, Norman Borloug was awarded the Nobel Prize and credited with saving over a billion people from starvation. In what is now called the Green Revolution, Borlaug led the research and development over a two-decade span beginning inthe 1940's to dramatically increases agricultural production worldwide. He introduced the synthetic farming methods already common in the United Stated and Britain to a global market,focusing particularly on the developing world and secceeded in hiking food production and saving lives. Borlauf's intiative calls for celebration.Yet,it is these same agricultural techniques-those associated with conventional farming-that have a bad over the last twenty-five years, causing the organic food market to a sour to a whopping $63 billion by 2012. These laborsaving,high-yeilding techniques bgan in the late 18th centuary and were perfected for nearly two centuries before worry spread that they seriously harmed the soil and allowed toxic chemicals to enter the food supply. In the 1940's while Borloug was busy feeding the world's impoverished, Albert and Gabrielle Howard-both accomplished botanists-were developing organic agriculture. Organic farming is the process by which crops are raised usnig only natural methods to maintain soil fertility and control pests.In the current food market, GMO's or genetically modified organisms, turn noses faster than saturated fats and soda pop. Instead,organic farmers rely on crop rotation, green manure and biological pest control, while excluding synthetic fertilizers,pesticides and growth hormones. Organic agriculture is said to promote sustainability, openness, health and safety and it standard are closely regulated by the International Federation of organic Agriculture MOvements.The IFOAM bases the foundation of organic farming on the minimal use of oo-farm inputs and on management practices that restore,preserve and improve ecological harmony. While this strategy sounds more "conventional" and effortless that what is now coined conventional farming, organic agriculture is actually quite scientific.Ecologically,organic farming is designed to promote and enhance biodiversity, so it must combine scientific knowledge and technologies to stimulate naturally occuring biological processes. For instance, organic farming uses pyrethrin, a natural pesticide found in the chrysanthemum flower, to deter pests and potassium bicarbonate to control diseases and suppress unruly weeds. Furthermore,where conventional farming focuses on mass production of each individual crop,organic farming encourages polyculture or multiple crops being raised in the same space . To replace nutrients, organic farming relies on the natural breakdown of organic matter by microorganisms like mycorrhiza, which forms a symbiotic relationship between fungi and plant roots. To relenish nitrogen, green manure is created by leaving uprooted crop parts to wither on a feild and is then used as a cover crop to fix nitrogen into the soil.The science doesn't stop with the crops. On farms with livestock, the feild of agroecology-which includes organic agriculture-attempts to provide animals with natural living conditions and feed. Just like in plants,organic farming rejects any growth hormones or genetic engineering in animals. The USDA has specific regulations in regard to organic livestock,demanding that the animals recieves only orgaic feed and are pastured rather than caged.Despite the popularity of organic foods, many argue that the concerns over conventional farming are a luxury of the rich. Organic farming yeild far less than conventional methods, uses more land and more labour, and is, therefore, more expensive.When prices rise and population falls, it is the poor that suffer. With the United Nations reporting 870 million people worldwide suffering from chronic malnutrition, organic farming faces a tough arguement against the capital-intensive, prolific conventional means. Science has a lot more work to do before organic agricultural methods can feed the world. Q. The author generally believes that synthetic agriculture is

Answer»

an INFERIOR alternative to the more sustanable organic agriculture.
the healthiest individual option for customers, both wealthy and poor.
a misunderstood potential solution to globe food shortages.
solidly grounded in science,UNLIKE organic agricultural techniques.

Solution :(C ) "Synthetic" is the opposite of organic, so it refers to "conventiomal" manmade approaches to farming. The author shows both PROS and cons of the synthetic approach,arguing that while it may not be as healthy or environmentally safe, it is much cheaper than the organic approach and can positively impact GLOBAL hunger. (A) is INCORRECT because the author does not believe that synthetic agriculture is inferior to organic. (B) is incorrect because organic is healthier, but not accessible to the poor. And (D) doesn't work because the passage argue that both methods are scientific.
182.

A Natural SyntheticIn 1970, Norman Borloug was awarded the Nobel Prize and credited with saving over a billion people from starvation. In what is now called the Green Revolution, Borlaug led the research and development over a two-decade span beginning inthe 1940's to dramatically increases agricultural production worldwide. He introduced the synthetic farming methods already common in the United Stated and Britain to a global market,focusing particularly on the developing world and secceeded in hiking food production and saving lives. Borlauf's intiative calls for celebration.Yet,it is these same agricultural techniques-those associated with conventional farming-that have a bad over the last twenty-five years, causing the organic food market to a sour to a whopping $63 billion by 2012. These laborsaving,high-yeilding techniques bgan in the late 18th centuary and were perfected for nearly two centuries before worry spread that they seriously harmed the soil and allowed toxic chemicals to enter the food supply. In the 1940's while Borloug was busy feeding the world's impoverished, Albert and Gabrielle Howard-both accomplished botanists-were developing organic agriculture. Organic farming is the process by which crops are raised usnig only natural methods to maintain soil fertility and control pests.In the current food market, GMO's or genetically modified organisms, turn noses faster than saturated fats and soda pop. Instead,organic farmers rely on crop rotation, green manure and biological pest control, while excluding synthetic fertilizers,pesticides and growth hormones. Organic agriculture is said to promote sustainability, openness, health and safety and it standard are closely regulated by the International Federation of organic Agriculture MOvements.The IFOAM bases the foundation of organic farming on the minimal use of oo-farm inputs and on management practices that restore,preserve and improve ecological harmony. While this strategy sounds more "conventional" and effortless that what is now coined conventional farming, organic agriculture is actually quite scientific.Ecologically,organic farming is designed to promote and enhance biodiversity, so it must combine scientific knowledge and technologies to stimulate naturally occuring biological processes. For instance, organic farming uses pyrethrin, a natural pesticide found in the chrysanthemum flower, to deter pests and potassium bicarbonate to control diseases and suppress unruly weeds. Furthermore,where conventional farming focuses on mass production of each individual crop,organic farming encourages polyculture or multiple crops being raised in the same space . To replace nutrients, organic farming relies on the natural breakdown of organic matter by microorganisms like mycorrhiza, which forms a symbiotic relationship between fungi and plant roots. To relenish nitrogen, green manure is created by leaving uprooted crop parts to wither on a feild and is then used as a cover crop to fix nitrogen into the soil.The science doesn't stop with the crops. On farms with livestock, the feild of agroecology-which includes organic agriculture-attempts to provide animals with natural living conditions and feed. Just like in plants,organic farming rejects any growth hormones or genetic engineering in animals. The USDA has specific regulations in regard to organic livestock,demanding that the animals recieves only orgaic feed and are pastured rather than caged.Despite the popularity of organic foods, many argue that the concerns over conventional farming are a luxury of the rich. Organic farming yeild far less than conventional methods, uses more land and more labour, and is, therefore, more expensive.When prices rise and population falls, it is the poor that suffer. With the United Nations reporting 870 million people worldwide suffering from chronic malnutrition, organic farming faces a tough arguement against the capital-intensive, prolific conventional means. Science has a lot more work to do before organic agricultural methods can feed the world. Q. Which option gives the best evidence for the answer to the previous question ?

Answer»

Lines 5-7 ("He introduced…lives")
Line 25-27 ("While this…scientific")
Lines 32-34 ("Furthermore…space")
Lines 45-47 ("Despite…expensive")

Solution :(C ) Lines 32-34 compare the individual focus of the conventional farming to the multi-crop environment of organic farming, MAKING it directly applicable to the previous question. (A) merely illustrates Borlaug's contrivution via the GREEN Revolution. (B) suggests that organic farming uses SCIENCE to mimic and provoke natural processes. (D) considers that the expense of organic agriculture doesn't ALLOW it a comprehensive market.
183.

Baking was a profitable profession in the old days in Goa. Prove it by giving examples from the text.

Answer»

Solution :Baking was indeed a profitable profession in the old days in Goa. The people of Goa were used to the refreshing fragrance of the loaves of bread. On all occasions and CEREMONIES, they NEEDED them. Marriage gifts were meaningless without the sweet bread or the bol. No party was complete if bread was not served in it. The lady of the house must prepare sandwiches on the engagement ceremony of her daughter. CHRISTMAS and other festivals must have bolinhas during their celebrations. The presence of the BAKER.s furnace was absolutely necessary in EVERY Goan village. The baker usually collected his bills at the end of the month. The baker and his family never starved. They always looked happy and prosperous. Their plump physique was an open testimony to their happiness and prosperity.
184.

As we …................. (a)…………………we could see the marshes …................. (b)…………………land …................. (c)…………………became dim in the mist …................. (d)…………………was gradually disappearing. We …................. (e)………………… to strain our eyes to see the slopes of the mountains beyond. The sky …................. (f)…………………reddish yellow. As the ship sailed along the white beaches we could …................. (g)…………………tiny voices …................. (h)…………………in a chorus.

Answer»

has
have
had
having

185.

Ausable did not fit any description of a secret agent. Comment.

Answer»

Solution :Ausable did not fit any description of a SECRET agent. He did not seem to be so smart, active, movable, aggressive and romantic as we READ in books or see in films. He was very fat and sloppy. Crack of pistols, WINE and dark-eyed beauties are normally associated with secret agents LIKE James Bond. These qualities had NOTHING to do with Ausable.
186.

Carolyn Wells takes liberties with language and employs humour to describe the wild animals. Give some examples of humorous descriptions in the poem.

Answer»

Solution :It depends on the subject matter what style a poet adopts while writing a poem. The poem DESCRIBES wild animals and how to distinguish them. To avoid oversimplification and monotony, the poet uses .humour. as an effective weapon. The .humour. used in the poem is suggestive and mild. It is not biting or scathing in effect. If the Asian Lion "roars at you as you.re dyin". The Bengal Tiger is a .noble. wild beast that "greets you” when you are roaming round in the jungle. The unsparing Leopard will show no mercy and it will “do no good to roar with pain.” If you have any doubts, the bear will "give you just one more CARESS." The poet uses subtle humour when he SAYS: "Hyenas come with merry smiles, But if they weep they.re CROCODILES".
187.

Anne says teachers are most unpredictable. Is Mr. Keesing unpredictable? How?

Answer»

Solution :ANNE is not wrong when she says that teachers are most unpredictable. It depends upon them WHETHER a student will go up in the next class or not. Mr. Keesing loses his temperament on TRIFLES. He goes on punishing Anne for her talking too MUCH by giving assignments of writing essays. But his ATTITUDE changes in the end. He allows her to talk and stops giving her extra work as punishment.
188.

Change the following sentences into Indirect Speech : Ramesh says, ''I have written a letter.''

Answer»

SOLUTION :RAMESH SAYS that he has WRITTEN a LETTER.
189.

B-26, Sector-20 Noida 18th March 20XX The General ManagerFull Marks Pvt Ltd. New DelhiSub: Application for the post of an Editor Dear Sir, With reference to your advertisement (a) underline ("on") 'The Times of India' dated 16th Mrch, 20XX, you (b) underline ("needs ")English Editors. Will you please consider (C)underline(" I") an applicant for the same position? I am postgraduate (d)underline(" from ")E lish Literature and possesses nearly five years of editing experience So, kindly give me an opportunity to face an interview with your esteemed organisation. Thanking you Yours sincerely Kapil Gupta

Answer»


Solution : B-26, Sector-20
Noida
18th March 20XX
The General Manager
Full MARKS Pvt Ltd.
New Delhi
Sub: Application for the post of an Editor
Dear Sir,
With reference to your advertisement (a) `underline (" in")` .The Times of India. dated 16TH Mrch, 20XX, you (b) `underline (" need ")`ENGLISH Editors. Will you please consider (C)`underline("me ")` an APPLICANT for the same position? I am postgraduate (d)`underline(" in ")`E lish Literature and possesses nearly five years of editing experience
So, kindly give me an opportunity to face an interview with your esteemed ORGANISATION. Thanking you
Yours sincerely
Kapil Gupta
190.

Below is the beginning of Harriet beecher stowe's 1852 novel. Uncle tom's cabin. Her own remark on te chapter is as follows: in which the redder isintroduced to a man of humanity. Late in the afternoon of a chilly day in february two genelemen were sitting over their wine.in a well furnished parlour in the line town of P--- in kentucky in the midst of anearnest conversation. That is the way i should arrange the matter, said Mr. shelby, the owner of the place. the fact is Tome is an uncommon fellow, he is certainly worth that sum anywheree steady, honest, capable, manges my farm lie a clock. You ought to let him cover the whole of the debt, and you would, haley, if you'd got any conscience. " Well, I've got just as much conscience as any man in business can afford to keep, siad haley, and I',m willing to do anything to blige friends, but this yer, ye see, is too hard on a feller, it really is. haven't you a boy or gal on a feller, it realyy in with tom? Hum-- none that I Could well spare, to tell te truth, it s only hard necessity makes me sell at all." Here the door opened, and a small quadroon boy, remarkably beauti-ful and engaging, entered with a comic air of assurance which showed e was used to being petted and noticed by his master. hulloa. Jim crow," said mr. shelby, snapping a bunch of raisins towards him, " pick that up, Now the child scampered, with all his little strength after the prze while his master laughed. tel you what siad haley, " Fling in that cahp, and I' ll settle the business, I wil. At this moment a young woma. obviously the child's mother, came in search of him, and haley, as soon as she had carried him away, turned to mr. Shelby in admiration. By jupiter said the trader, there's an article now you might make your fortune on that one gal in orleans, any way what shall I say for her? What'll you take? " Mr haley, she is not to be sold. I say no, and I mean no," said Mr. shelby. decidedly. Well. you'll let me have the boy, though i would rather not sell him, '' said Mr. i would rather not sell him. Said mr' Shelby, the fact is I' m a humane man, and I hate to take the boy from his mother, sir. Oh, you do? la, Yes, I understand per-factly. It is mighty unpleasant getting on fectly. It is mighty unpleasant getting on with women sometimes. I al' ays hates these with women sometimes. I al' as hates these with women sometimes. I al' ays hates these yer screechin times. As I manages business. I generally avoids em, sir. now, what if you get the gal off for a day or so? then the thing's done quietly. it 's always best to do the humane thing. sir, that's been my experience." "I'd lie to have been able to kick the fellow down the steps, siad mr. shelby to himself, when the trader had bowed himself himself, when the trader had bowed humself out. And Eliza's child. too I know I shal havesome fuss with the wife about that, and for that matter, about tom, too so much for being in dibt, heigho the prayer-meeting at uncle tome's cabin had been protracted to a very late hour, and Tom and his worthy helpmeet were not yet asleep, when between twelve and one there was a light tap on the window pane good Lord what that said Aunt chloe, starting up. My sakes alive it it aint Lizzy Get on your clothes, old man, quick. I'm gwine to open the door. and suiting the action to the word, the door flew open and the light of the candle wich Tom had hastily lighted. fell on the face of elizxa. I' m running away. Uncle tom and aunt chloe__ carrying off my child. Master sold him. " " Sold him? echoed both. holding up their hands in dismay. " Yes. sold him said eliza firmly. ' I crept into ther closet by mistre4ss's door to night, and I heard master tel missus that he had sold my harry and you , Uncle tom, both to a trader, and that the man was to take posses- sion to - day" Slowly, as the meaning of this speech came over Tom, he collapsed on his old chair, and sunk his head on his knees. 3. As used in line 31, the pharase fling in most closely means

Answer»

INCLUDE
hurl
RELATE
involve

Solution :Include could be a substitute for fling in since this sentence REFERS to HALEY's request that Mr. Shelby add the dmall boy to the deal. Hurl would incorrectly indicate the act of THROWING. Involve is too weak for this context. And it would be nonsensical to say that haey wanted Mr. shelby to relate the boy.
191.

Below is the beginning of Harriet beecher stowe's 1852 novel. Uncle tom's cabin. Her own remark on te chapter is as follows: in which the redder isintroduced to a man of humanity. Late in the afternoon of a chilly day in february two genelemen were sitting over their wine.in a well furnished parlour in the line town of P--- in kentucky in the midst of anearnest conversation. That is the way i should arrange the matter, said Mr. shelby, the owner of the place. the fact is Tome is an uncommon fellow, he is certainly worth that sum anywheree steady, honest, capable, manges my farm lie a clock. You ought to let him cover the whole of the debt, and you would, haley, if you'd got any conscience. " Well, I've got just as much conscience as any man in business can afford to keep, siad haley, and I',m willing to do anything to blige friends, but this yer, ye see, is too hard on a feller, it really is. haven't you a boy or gal on a feller, it realyy in with tom? Hum-- none that I Could well spare, to tell te truth, it s only hard necessity makes me sell at all." Here the door opened, and a small quadroon boy, remarkably beauti-ful and engaging, entered with a comic air of assurance which showed e was used to being petted and noticed by his master. hulloa. Jim crow," said mr. shelby, snapping a bunch of raisins towards him, " pick that up, Now the child scampered, with all his little strength after the prze while his master laughed. tel you what siad haley, " Fling in that cahp, and I' ll settle the business, I wil. At this moment a young woma. obviously the child's mother, came in search of him, and haley, as soon as she had carried him away, turned to mr. Shelby in admiration. By jupiter said the trader, there's an article now you might make your fortune on that one gal in orleans, any way what shall I say for her? What'll you take? " Mr haley, she is not to be sold. I say no, and I mean no," said Mr. shelby. decidedly. Well. you'll let me have the boy, though i would rather not sell him, '' said Mr. i would rather not sell him. Said mr' Shelby, the fact is I' m a humane man, and I hate to take the boy from his mother, sir. Oh, you do? la, Yes, I understand per-factly. It is mighty unpleasant getting on fectly. It is mighty unpleasant getting on with women sometimes. I al' ays hates these with women sometimes. I al' as hates these with women sometimes. I al' ays hates these yer screechin times. As I manages business. I generally avoids em, sir. now, what if you get the gal off for a day or so? then the thing's done quietly. it 's always best to do the humane thing. sir, that's been my experience." "I'd lie to have been able to kick the fellow down the steps, siad mr. shelby to himself, when the trader had bowed himself himself, when the trader had bowed humself out. And Eliza's child. too I know I shal havesome fuss with the wife about that, and for that matter, about tom, too so much for being in dibt, heigho the prayer-meeting at uncle tome's cabin had been protracted to a very late hour, and Tom and his worthy helpmeet were not yet asleep, when between twelve and one there was a light tap on the window pane good Lord what that said Aunt chloe, starting up. My sakes alive it it aint Lizzy Get on your clothes, old man, quick. I'm gwine to open the door. and suiting the action to the word, the door flew open and the light of the candle wich Tom had hastily lighted. fell on the face of elizxa. I' m running away. Uncle tom and aunt chloe__ carrying off my child. Master sold him. " " Sold him? echoed both. holding up their hands in dismay. " Yes. sold him said eliza firmly. ' I crept into ther closet by mistre4ss's door to night, and I heard master tel missus that he had sold my harry and you , Uncle tom, both to a trader, and that the man was to take posses- sion to - day" Slowly, as the meaning of this speech came over Tom, he collapsed on his old chair, and sunk his head on his knees. 4. Mr. shelby's tratment of the ch9ld in lines 27-32 3. As used in line 31, the phrase " fling in " most closely means

Answer»

purposel dceitful
unitentionally inhumane.
openly belligerent
tendeerly impartial.

SOLUTION :There is evidence in the passage that mr. Shelby is RELUCTANT to sell his slaves, and doesn’t want to separate the child from his mother. Yet, his treatment of the boy in these particular lines is more fitting for an ADORED pet than a human child, making choice(B) correct. The passage does not SUPPORT the IDEA that Mr. shelby is dishnest or aggressive. and finally, choice (D ) is contradictive since impartical means" neutral or disinterested. "
192.

Below is the beginning of Harriet beecher stowe's 1852 novel. Uncle tom's cabin. Her own remark on te chapter is as follows: in which the redder isintroduced to a man of humanity. Late in the afternoon of a chilly day in february two genelemen were sitting over their wine.in a well furnished parlour in the line town of P--- in kentucky in the midst of anearnest conversation. That is the way i should arrange the matter, said Mr. shelby, the owner of the place. the fact is Tome is an uncommon fellow, he is certainly worth that sum anywheree steady, honest, capable, manges my farm lie a clock. You ought to let him cover the whole of the debt, and you would, haley, if you'd got any conscience. " Well, I've got just as much conscience as any man in business can afford to keep, siad haley, and I',m willing to do anything to blige friends, but this yer, ye see, is too hard on a feller, it really is. haven't you a boy or gal on a feller, it realyy in with tom? Hum-- none that I Could well spare, to tell te truth, it s only hard necessity makes me sell at all." Here the door opened, and a small quadroon boy, remarkably beauti-ful and engaging, entered with a comic air of assurance which showed e was used to being petted and noticed by his master. hulloa. Jim crow," said mr. shelby, snapping a bunch of raisins towards him, " pick that up, Now the child scampered, with all his little strength after the prze while his master laughed. tel you what siad haley, " Fling in that cahp, and I' ll settle the business, I wil. At this moment a young woma. obviously the child's mother, came in search of him, and haley, as soon as she had carried him away, turned to mr. Shelby in admiration. By jupiter said the trader, there's an article now you might make your fortune on that one gal in orleans, any way what shall I say for her? What'll you take? " Mr haley, she is not to be sold. I say no, and I mean no," said Mr. shelby. decidedly. Well. you'll let me have the boy, though i would rather not sell him, '' said Mr. i would rather not sell him. Said mr' Shelby, the fact is I' m a humane man, and I hate to take the boy from his mother, sir. Oh, you do? la, Yes, I understand per-factly. It is mighty unpleasant getting on fectly. It is mighty unpleasant getting on with women sometimes. I al' ays hates these with women sometimes. I al' as hates these with women sometimes. I al' ays hates these yer screechin times. As I manages business. I generally avoids em, sir. now, what if you get the gal off for a day or so? then the thing's done quietly. it 's always best to do the humane thing. sir, that's been my experience." "I'd lie to have been able to kick the fellow down the steps, siad mr. shelby to himself, when the trader had bowed himself himself, when the trader had bowed humself out. And Eliza's child. too I know I shal havesome fuss with the wife about that, and for that matter, about tom, too so much for being in dibt, heigho the prayer-meeting at uncle tome's cabin had been protracted to a very late hour, and Tom and his worthy helpmeet were not yet asleep, when between twelve and one there was a light tap on the window pane good Lord what that said Aunt chloe, starting up. My sakes alive it it aint Lizzy Get on your clothes, old man, quick. I'm gwine to open the door. and suiting the action to the word, the door flew open and the light of the candle wich Tom had hastily lighted. fell on the face of elizxa. I' m running away. Uncle tom and aunt chloe__ carrying off my child. Master sold him. " " Sold him? echoed both. holding up their hands in dismay. " Yes. sold him said eliza firmly. ' I crept into ther closet by mistre4ss's door to night, and I heard master tel missus that he had sold my harry and you , Uncle tom, both to a trader, and that the man was to take posses- sion to - day" Slowly, as the meaning of this speech came over Tom, he collapsed on his old chair, and sunk his head on his knees. 5.As used in line 38, the word" article" most closely means

Answer»

agreement
report
word
ITEM.

SOLUTION :Item most closely fits the meaning of article in this line, SINCE it refers to eliza as an object. (A), (B) and (C0 are not fitting since they do not rer to the woman as a commodity as INTENDED by the speaker.
193.

Below is the beginning of Harriet beecher stowe's 1852 novel. Uncle tom's cabin. Her own remark on te chapter is as follows: in which the redder isintroduced to a man of humanity. Late in the afternoon of a chilly day in february two genelemen were sitting over their wine.in a well furnished parlour in the line town of P--- in kentucky in the midst of anearnest conversation. That is the way i should arrange the matter, said Mr. shelby, the owner of the place. the fact is Tome is an uncommon fellow, he is certainly worth that sum anywheree steady, honest, capable, manges my farm lie a clock. You ought to let him cover the whole of the debt, and you would, haley, if you'd got any conscience. " Well, I've got just as much conscience as any man in business can afford to keep, siad haley, and I',m willing to do anything to blige friends, but this yer, ye see, is too hard on a feller, it really is. haven't you a boy or gal on a feller, it realyy in with tom? Hum-- none that I Could well spare, to tell te truth, it s only hard necessity makes me sell at all." Here the door opened, and a small quadroon boy, remarkably beauti-ful and engaging, entered with a comic air of assurance which showed e was used to being petted and noticed by his master. hulloa. Jim crow," said mr. shelby, snapping a bunch of raisins towards him, " pick that up, Now the child scampered, with all his little strength after the prze while his master laughed. tel you what siad haley, " Fling in that cahp, and I' ll settle the business, I wil. At this moment a young woma. obviously the child's mother, came in search of him, and haley, as soon as she had carried him away, turned to mr. Shelby in admiration. By jupiter said the trader, there's an article now you might make your fortune on that one gal in orleans, any way what shall I say for her? What'll you take? " Mr haley, she is not to be sold. I say no, and I mean no," said Mr. shelby. decidedly. Well. you'll let me have the boy, though i would rather not sell him, '' said Mr. i would rather not sell him. Said mr' Shelby, the fact is I' m a humane man, and I hate to take the boy from his mother, sir. Oh, you do? la, Yes, I understand per-factly. It is mighty unpleasant getting on fectly. It is mighty unpleasant getting on with women sometimes. I al' ays hates these with women sometimes. I al' as hates these with women sometimes. I al' ays hates these yer screechin times. As I manages business. I generally avoids em, sir. now, what if you get the gal off for a day or so? then the thing's done quietly. it 's always best to do the humane thing. sir, that's been my experience." "I'd lie to have been able to kick the fellow down the steps, siad mr. shelby to himself, when the trader had bowed himself himself, when the trader had bowed humself out. And Eliza's child. too I know I shal havesome fuss with the wife about that, and for that matter, about tom, too so much for being in dibt, heigho the prayer-meeting at uncle tome's cabin had been protracted to a very late hour, and Tom and his worthy helpmeet were not yet asleep, when between twelve and one there was a light tap on the window pane good Lord what that said Aunt chloe, starting up. My sakes alive it it aint Lizzy Get on your clothes, old man, quick. I'm gwine to open the door. and suiting the action to the word, the door flew open and the light of the candle wich Tom had hastily lighted. fell on the face of elizxa. I' m running away. Uncle tom and aunt chloe__ carrying off my child. Master sold him. " " Sold him? echoed both. holding up their hands in dismay. " Yes. sold him said eliza firmly. ' I crept into ther closet by mistre4ss's door to night, and I heard master tel missus that he had sold my harry and you , Uncle tom, both to a trader, and that the man was to take posses- sion to - day" Slowly, as the meaning of this speech came over Tom, he collapsed on his old chair, and sunk his head on his knees. 6. the passage most sterongley implies that tom's reaction to hearing of mr. shelby's plans for him is one of

Answer»

unanticipated peacefulness.
delighted RELIEF.
surprised despondency
playful mockery.

Solution :Lines 84-86 INDICATE that UPON HEARING the news, tom is SHOCKED and jdejected. Despondencyrefers to hopelessness and joylessness. His reaction is neither peaceful. Relieved. Nor playful so (A) , (B ) and (D) are incorrect.
194.

Below is the beginning of Harriet beecher stowe's 1852 novel. Uncle tom's cabin. Her own remark on te chapter is as follows: in which the redder isintroduced to a man of humanity. Late in the afternoon of a chilly day in february two genelemen were sitting over their wine.in a well furnished parlour in the line town of P--- in kentucky in the midst of anearnest conversation. That is the way i should arrange the matter, said Mr. shelby, the owner of the place. the fact is Tome is an uncommon fellow, he is certainly worth that sum anywheree steady, honest, capable, manges my farm lie a clock. You ought to let him cover the whole of the debt, and you would, haley, if you'd got any conscience. " Well, I've got just as much conscience as any man in business can afford to keep, siad haley, and I',m willing to do anything to blige friends, but this yer, ye see, is too hard on a feller, it really is. haven't you a boy or gal on a feller, it realyy in with tom? Hum-- none that I Could well spare, to tell te truth, it s only hard necessity makes me sell at all." Here the door opened, and a small quadroon boy, remarkably beauti-ful and engaging, entered with a comic air of assurance which showed e was used to being petted and noticed by his master. hulloa. Jim crow," said mr. shelby, snapping a bunch of raisins towards him, " pick that up, Now the child scampered, with all his little strength after the prze while his master laughed. tel you what siad haley, " Fling in that cahp, and I' ll settle the business, I wil. At this moment a young woma. obviously the child's mother, came in search of him, and haley, as soon as she had carried him away, turned to mr. Shelby in admiration. By jupiter said the trader, there's an article now you might make your fortune on that one gal in orleans, any way what shall I say for her? What'll you take? " Mr haley, she is not to be sold. I say no, and I mean no," said Mr. shelby. decidedly. Well. you'll let me have the boy, though i would rather not sell him, '' said Mr. i would rather not sell him. Said mr' Shelby, the fact is I' m a humane man, and I hate to take the boy from his mother, sir. Oh, you do? la, Yes, I understand per-factly. It is mighty unpleasant getting on fectly. It is mighty unpleasant getting on with women sometimes. I al' ays hates these with women sometimes. I al' as hates these with women sometimes. I al' ays hates these yer screechin times. As I manages business. I generally avoids em, sir. now, what if you get the gal off for a day or so? then the thing's done quietly. it 's always best to do the humane thing. sir, that's been my experience." "I'd lie to have been able to kick the fellow down the steps, siad mr. shelby to himself, when the trader had bowed himself himself, when the trader had bowed humself out. And Eliza's child. too I know I shal havesome fuss with the wife about that, and for that matter, about tom, too so much for being in dibt, heigho the prayer-meeting at uncle tome's cabin had been protracted to a very late hour, and Tom and his worthy helpmeet were not yet asleep, when between twelve and one there was a light tap on the window pane good Lord what that said Aunt chloe, starting up. My sakes alive it it aint Lizzy Get on your clothes, old man, quick. I'm gwine to open the door. and suiting the action to the word, the door flew open and the light of the candle wich Tom had hastily lighted. fell on the face of elizxa. I' m running away. Uncle tom and aunt chloe__ carrying off my child. Master sold him. " " Sold him? echoed both. holding up their hands in dismay. " Yes. sold him said eliza firmly. ' I crept into ther closet by mistre4ss's door to night, and I heard master tel missus that he had sold my harry and you , Uncle tom, both to a trader, and that the man was to take posses- sion to - day" Slowly, as the meaning of this speech came over Tom, he collapsed on his old chair, and sunk his head on his knees. 1. Which choice provides the best summary of what happened in the passage?

Answer»

A deal is reluctantly made and the ractions of those affected are given.
A slave SUCCESSFULLY plots an escape from an oppressive society.
A man struggles to choose between what is humane and what is profitable.
A PHILOSOPHICAL discussion is held between a slave-owner and a slave-trader.

Solution :this passage can be paraphrased best by choice (A) . (B) is not supportd since the running away has not yet happened . (c) is tempting. But ULTI ately incorrect because mr. shelby is not trying to kmake a profit, but to PAY off a debt. (D) is a detail but not a summary.
195.

Below is the beginning of Harriet beecher stowe's 1852 novel. Uncle tom's cabin. Her own remark on te chapter is as follows: in which the redder isintroduced to a man of humanity. Late in the afternoon of a chilly day in february two genelemen were sitting over their wine.in a well furnished parlour in the line town of P--- in kentucky in the midst of anearnest conversation. That is the way i should arrange the matter, said Mr. shelby, the owner of the place. the fact is Tome is an uncommon fellow, he is certainly worth that sum anywheree steady, honest, capable, manges my farm lie a clock. You ought to let him cover the whole of the debt, and you would, haley, if you'd got any conscience. " Well, I've got just as much conscience as any man in business can afford to keep, siad haley, and I',m willing to do anything to blige friends, but this yer, ye see, is too hard on a feller, it really is. haven't you a boy or gal on a feller, it realyy in with tom? Hum-- none that I Could well spare, to tell te truth, it s only hard necessity makes me sell at all." Here the door opened, and a small quadroon boy, remarkably beauti-ful and engaging, entered with a comic air of assurance which showed e was used to being petted and noticed by his master. hulloa. Jim crow," said mr. shelby, snapping a bunch of raisins towards him, " pick that up, Now the child scampered, with all his little strength after the prze while his master laughed. tel you what siad haley, " Fling in that cahp, and I' ll settle the business, I wil. At this moment a young woma. obviously the child's mother, came in search of him, and haley, as soon as she had carried him away, turned to mr. Shelby in admiration. By jupiter said the trader, there's an article now you might make your fortune on that one gal in orleans, any way what shall I say for her? What'll you take? " Mr haley, she is not to be sold. I say no, and I mean no," said Mr. shelby. decidedly. Well. you'll let me have the boy, though i would rather not sell him, '' said Mr. i would rather not sell him. Said mr' Shelby, the fact is I' m a humane man, and I hate to take the boy from his mother, sir. Oh, you do? la, Yes, I understand per-factly. It is mighty unpleasant getting on fectly. It is mighty unpleasant getting on with women sometimes. I al' ays hates these with women sometimes. I al' as hates these with women sometimes. I al' ays hates these yer screechin times. As I manages business. I generally avoids em, sir. now, what if you get the gal off for a day or so? then the thing's done quietly. it 's always best to do the humane thing. sir, that's been my experience." "I'd lie to have been able to kick the fellow down the steps, siad mr. shelby to himself, when the trader had bowed himself himself, when the trader had bowed humself out. And Eliza's child. too I know I shal havesome fuss with the wife about that, and for that matter, about tom, too so much for being in dibt, heigho the prayer-meeting at uncle tome's cabin had been protracted to a very late hour, and Tom and his worthy helpmeet were not yet asleep, when between twelve and one there was a light tap on the window pane good Lord what that said Aunt chloe, starting up. My sakes alive it it aint Lizzy Get on your clothes, old man, quick. I'm gwine to open the door. and suiting the action to the word, the door flew open and the light of the candle wich Tom had hastily lighted. fell on the face of elizxa. I' m running away. Uncle tom and aunt chloe__ carrying off my child. Master sold him. " " Sold him? echoed both. holding up their hands in dismay. " Yes. sold him said eliza firmly. ' I crept into ther closet by mistre4ss's door to night, and I heard master tel missus that he had sold my harry and you , Uncle tom, both to a trader, and that the man was to take posses- sion to - day" Slowly, as the meaning of this speech came over Tom, he collapsed on his old chair, and sunk his head on his knees. 2. Haley is best cahracterizedas a/an

Answer»

HUMANE empathizer
financial amateur
aggressive negotiator
passive mediator

Solution :From the conversation between haely and Mr. SHELBY. It is clear that heley is first and formost a businessman. Moreover, he won't bhe persuaded to just accept tom as payment FO the bebt, so (c) is an ACCURATE DEPICTION of his character . He is neither empathizing nor passive. In fact, he is nearly incompromising. Finally. choice (B) won't work because we can infer by hs negotiation skills that he is not an amateur.
196.

Andrew couldn't help it. He was still harboring bad feelings about his ex-girlfreind long after breakup. There was simply no excuse for how she had treated him.

Answer»

disregarding
camouflaging entertaining
cherishing

Solution :(C ) This ONE can be tricky. (C ) is correct because this usage of enternaning means "THINKING of OFTEN or deeply". Choose (B) does not work since it is unlikely that he would have the self-control to keep these feelings secrect. Choices (A) and (D) are the opposite of what the senteces might CONVEY.
197.

Below is the beginning of Harriet beecher stowe's 1852 novel. Uncle tom's cabin. Her own remark on te chapter is as follows: in which the redder isintroduced to a man of humanity. Late in the afternoon of a chilly day in february two genelemen were sitting over their wine.in a well furnished parlour in the line town of P--- in kentucky in the midst of anearnest conversation. That is the way i should arrange the matter, said Mr. shelby, the owner of the place. the fact is Tome is an uncommon fellow, he is certainly worth that sum anywheree steady, honest, capable, manges my farm lie a clock. You ought to let him cover the whole of the debt, and you would, haley, if you'd got any conscience. " Well, I've got just as much conscience as any man in business can afford to keep, siad haley, and I',m willing to do anything to blige friends, but this yer, ye see, is too hard on a feller, it really is. haven't you a boy or gal on a feller, it realyy in with tom? Hum-- none that I Could well spare, to tell te truth, it s only hard necessity makes me sell at all." Here the door opened, and a small quadroon boy, remarkably beauti-ful and engaging, entered with a comic air of assurance which showed e was used to being petted and noticed by his master. hulloa. Jim crow," said mr. shelby, snapping a bunch of raisins towards him, " pick that up, Now the child scampered, with all his little strength after the prze while his master laughed. tel you what siad haley, " Fling in that cahp, and I' ll settle the business, I wil. At this moment a young woma. obviously the child's mother, came in search of him, and haley, as soon as she had carried him away, turned to mr. Shelby in admiration. By jupiter said the trader, there's an article now you might make your fortune on that one gal in orleans, any way what shall I say for her? What'll you take? " Mr haley, she is not to be sold. I say no, and I mean no," said Mr. shelby. decidedly. Well. you'll let me have the boy, though i would rather not sell him, '' said Mr. i would rather not sell him. Said mr' Shelby, the fact is I' m a humane man, and I hate to take the boy from his mother, sir. Oh, you do? la, Yes, I understand per-factly. It is mighty unpleasant getting on fectly. It is mighty unpleasant getting on with women sometimes. I al' ays hates these with women sometimes. I al' as hates these with women sometimes. I al' ays hates these yer screechin times. As I manages business. I generally avoids em, sir. now, what if you get the gal off for a day or so? then the thing's done quietly. it 's always best to do the humane thing. sir, that's been my experience." "I'd lie to have been able to kick the fellow down the steps, siad mr. shelby to himself, when the trader had bowed himself himself, when the trader had bowed humself out. And Eliza's child. too I know I shal havesome fuss with the wife about that, and for that matter, about tom, too so much for being in dibt, heigho the prayer-meeting at uncle tome's cabin had been protracted to a very late hour, and Tom and his worthy helpmeet were not yet asleep, when between twelve and one there was a light tap on the window pane good Lord what that said Aunt chloe, starting up. My sakes alive it it aint Lizzy Get on your clothes, old man, quick. I'm gwine to open the door. and suiting the action to the word, the door flew open and the light of the candle wich Tom had hastily lighted. fell on the face of elizxa. I' m running away. Uncle tom and aunt chloe__ carrying off my child. Master sold him. " " Sold him? echoed both. holding up their hands in dismay. " Yes. sold him said eliza firmly. ' I crept into ther closet by mistre4ss's door to night, and I heard master tel missus that he had sold my harry and you , Uncle tom, both to a trader, and that the man was to take posses- sion to - day" Slowly, as the meaning of this speech came over Tom, he collapsed on his old chair, and sunk his head on his knees. 9. Which option gives the best evidence for the answer the previous question?

Answer»

Lines 8-13("the fact . . . Conscience" )
Lines 27-32 (Hulloa . . . Will")
Lines 37-42(" By . . . Decidedly")
Lines 67-70(" Good. . . Door" )

Solution :Lines 37-42 give the most direct evidence of Mr. Shelby's determination to keep Eliza regardless of what haley MIGHT offer. Choice (A) suggests a slight fonness for Tom, and choice (B) exhibits a sadistic affection for the CHILD, but in both cases, mrgt Shelby still agrees to SELL them. (D) is irrelevennt as it just reveals aunt chloe's SURPRISE at Eliza showing up at night unannounced.
198.

Below is the beginning of Harriet beecher stowe's 1852 novel. Uncle tom's cabin. Her own remark on te chapter is as follows: in which the redder isintroduced to a man of humanity. Late in the afternoon of a chilly day in february two genelemen were sitting over their wine.in a well furnished parlour in the line town of P--- in kentucky in the midst of anearnest conversation. That is the way i should arrange the matter, said Mr. shelby, the owner of the place. the fact is Tome is an uncommon fellow, he is certainly worth that sum anywheree steady, honest, capable, manges my farm lie a clock. You ought to let him cover the whole of the debt, and you would, haley, if you'd got any conscience. " Well, I've got just as much conscience as any man in business can afford to keep, siad haley, and I',m willing to do anything to blige friends, but this yer, ye see, is too hard on a feller, it really is. haven't you a boy or gal on a feller, it realyy in with tom? Hum-- none that I Could well spare, to tell te truth, it s only hard necessity makes me sell at all." Here the door opened, and a small quadroon boy, remarkably beauti-ful and engaging, entered with a comic air of assurance which showed e was used to being petted and noticed by his master. hulloa. Jim crow," said mr. shelby, snapping a bunch of raisins towards him, " pick that up, Now the child scampered, with all his little strength after the prze while his master laughed. tel you what siad haley, " Fling in that cahp, and I' ll settle the business, I wil. At this moment a young woma. obviously the child's mother, came in search of him, and haley, as soon as she had carried him away, turned to mr. Shelby in admiration. By jupiter said the trader, there's an article now you might make your fortune on that one gal in orleans, any way what shall I say for her? What'll you take? " Mr haley, she is not to be sold. I say no, and I mean no," said Mr. shelby. decidedly. Well. you'll let me have the boy, though i would rather not sell him, '' said Mr. i would rather not sell him. Said mr' Shelby, the fact is I' m a humane man, and I hate to take the boy from his mother, sir. Oh, you do? la, Yes, I understand per-factly. It is mighty unpleasant getting on fectly. It is mighty unpleasant getting on with women sometimes. I al' ays hates these with women sometimes. I al' as hates these with women sometimes. I al' ays hates these yer screechin times. As I manages business. I generally avoids em, sir. now, what if you get the gal off for a day or so? then the thing's done quietly. it 's always best to do the humane thing. sir, that's been my experience." "I'd lie to have been able to kick the fellow down the steps, siad mr. shelby to himself, when the trader had bowed himself himself, when the trader had bowed humself out. And Eliza's child. too I know I shal havesome fuss with the wife about that, and for that matter, about tom, too so much for being in dibt, heigho the prayer-meeting at uncle tome's cabin had been protracted to a very late hour, and Tom and his worthy helpmeet were not yet asleep, when between twelve and one there was a light tap on the window pane good Lord what that said Aunt chloe, starting up. My sakes alive it it aint Lizzy Get on your clothes, old man, quick. I'm gwine to open the door. and suiting the action to the word, the door flew open and the light of the candle wich Tom had hastily lighted. fell on the face of elizxa. I' m running away. Uncle tom and aunt chloe__ carrying off my child. Master sold him. " " Sold him? echoed both. holding up their hands in dismay. " Yes. sold him said eliza firmly. ' I crept into ther closet by mistre4ss's door to night, and I heard master tel missus that he had sold my harry and you , Uncle tom, both to a trader, and that the man was to take posses- sion to - day" Slowly, as the meaning of this speech came over Tom, he collapsed on his old chair, and sunk his head on his knees. 10. The " light tap " made by Eliza in line 66 suggests that she

Answer»

feared unwanted detection
respected nightly rituals
was HESITANT to share BAD news
undrstood her misdeeds

Solution :It is safe to assume that the light tap is indicatve of Eliza's discretion, as she must be very secretive about her plans to run away. So, it is fitting to SAY that she is TRYING to avoid detection as in CHOICE (A) . (C )is incorrect because we know she risked visiting Tom specifically to share the news. choice (B) wrongly assumes her caution is related to offending Tom and chloe by vivsiting them so late. Finally, choice (D) inaccurately assumes she is worried more about breaking the law than being caught.
199.

Below is the beginning of Harriet beecher stowe's 1852 novel. Uncle tom's cabin. Her own remark on te chapter is as follows: in which the redder isintroduced to a man of humanity. Late in the afternoon of a chilly day in february two genelemen were sitting over their wine.in a well furnished parlour in the line town of P--- in kentucky in the midst of anearnest conversation. That is the way i should arrange the matter, said Mr. shelby, the owner of the place. the fact is Tome is an uncommon fellow, he is certainly worth that sum anywheree steady, honest, capable, manges my farm lie a clock. You ought to let him cover the whole of the debt, and you would, haley, if you'd got any conscience. " Well, I've got just as much conscience as any man in business can afford to keep, siad haley, and I',m willing to do anything to blige friends, but this yer, ye see, is too hard on a feller, it really is. haven't you a boy or gal on a feller, it realyy in with tom? Hum-- none that I Could well spare, to tell te truth, it s only hard necessity makes me sell at all." Here the door opened, and a small quadroon boy, remarkably beauti-ful and engaging, entered with a comic air of assurance which showed e was used to being petted and noticed by his master. hulloa. Jim crow," said mr. shelby, snapping a bunch of raisins towards him, " pick that up, Now the child scampered, with all his little strength after the prze while his master laughed. tel you what siad haley, " Fling in that cahp, and I' ll settle the business, I wil. At this moment a young woma. obviously the child's mother, came in search of him, and haley, as soon as she had carried him away, turned to mr. Shelby in admiration. By jupiter said the trader, there's an article now you might make your fortune on that one gal in orleans, any way what shall I say for her? What'll you take? " Mr haley, she is not to be sold. I say no, and I mean no," said Mr. shelby. decidedly. Well. you'll let me have the boy, though i would rather not sell him, '' said Mr. i would rather not sell him. Said mr' Shelby, the fact is I' m a humane man, and I hate to take the boy from his mother, sir. Oh, you do? la, Yes, I understand per-factly. It is mighty unpleasant getting on fectly. It is mighty unpleasant getting on with women sometimes. I al' ays hates these with women sometimes. I al' as hates these with women sometimes. I al' ays hates these yer screechin times. As I manages business. I generally avoids em, sir. now, what if you get the gal off for a day or so? then the thing's done quietly. it 's always best to do the humane thing. sir, that's been my experience." "I'd lie to have been able to kick the fellow down the steps, siad mr. shelby to himself, when the trader had bowed himself himself, when the trader had bowed humself out. And Eliza's child. too I know I shal havesome fuss with the wife about that, and for that matter, about tom, too so much for being in dibt, heigho the prayer-meeting at uncle tome's cabin had been protracted to a very late hour, and Tom and his worthy helpmeet were not yet asleep, when between twelve and one there was a light tap on the window pane good Lord what that said Aunt chloe, starting up. My sakes alive it it aint Lizzy Get on your clothes, old man, quick. I'm gwine to open the door. and suiting the action to the word, the door flew open and the light of the candle wich Tom had hastily lighted. fell on the face of elizxa. I' m running away. Uncle tom and aunt chloe__ carrying off my child. Master sold him. " " Sold him? echoed both. holding up their hands in dismay. " Yes. sold him said eliza firmly. ' I crept into ther closet by mistre4ss's door to night, and I heard master tel missus that he had sold my harry and you , Uncle tom, both to a trader, and that the man was to take posses- sion to - day" Slowly, as the meaning of this speech came over Tom, he collapsed on his old chair, and sunk his head on his knees. 7 . Which option gives the best evidence for the answer to the previous question?

Answer»

Lines 58-61(" I know… heigho")
Lines 62-66( the prayer. . . Pane")
Lines 73-75( " I'm . . . Him)
Lines 84-86(slowly . . . Knees')

Solution :Tom" collapsed" and " sunk his head," so these lines PROVIDE DIRECT evidence for theprevious questiongt (A) and (c)refer to mr. Shelby's wife's and Elizas REACTIONS, respectively. And (B ) DESCRIBES Tom's state fefor hearing the news, RATHER than fafter.
200.

Below is the beginning of Harriet beecher stowe's 1852 novel. Uncle tom's cabin. Her own remark on te chapter is as follows: in which the redder isintroduced to a man of humanity. Late in the afternoon of a chilly day in february two genelemen were sitting over their wine.in a well furnished parlour in the line town of P--- in kentucky in the midst of anearnest conversation. That is the way i should arrange the matter, said Mr. shelby, the owner of the place. the fact is Tome is an uncommon fellow, he is certainly worth that sum anywheree steady, honest, capable, manges my farm lie a clock. You ought to let him cover the whole of the debt, and you would, haley, if you'd got any conscience. " Well, I've got just as much conscience as any man in business can afford to keep, siad haley, and I',m willing to do anything to blige friends, but this yer, ye see, is too hard on a feller, it really is. haven't you a boy or gal on a feller, it realyy in with tom? Hum-- none that I Could well spare, to tell te truth, it s only hard necessity makes me sell at all." Here the door opened, and a small quadroon boy, remarkably beauti-ful and engaging, entered with a comic air of assurance which showed e was used to being petted and noticed by his master. hulloa. Jim crow," said mr. shelby, snapping a bunch of raisins towards him, " pick that up, Now the child scampered, with all his little strength after the prze while his master laughed. tel you what siad haley, " Fling in that cahp, and I' ll settle the business, I wil. At this moment a young woma. obviously the child's mother, came in search of him, and haley, as soon as she had carried him away, turned to mr. Shelby in admiration. By jupiter said the trader, there's an article now you might make your fortune on that one gal in orleans, any way what shall I say for her? What'll you take? " Mr haley, she is not to be sold. I say no, and I mean no," said Mr. shelby. decidedly. Well. you'll let me have the boy, though i would rather not sell him, '' said Mr. i would rather not sell him. Said mr' Shelby, the fact is I' m a humane man, and I hate to take the boy from his mother, sir. Oh, you do? la, Yes, I understand per-factly. It is mighty unpleasant getting on fectly. It is mighty unpleasant getting on with women sometimes. I al' ays hates these with women sometimes. I al' as hates these with women sometimes. I al' ays hates these yer screechin times. As I manages business. I generally avoids em, sir. now, what if you get the gal off for a day or so? then the thing's done quietly. it 's always best to do the humane thing. sir, that's been my experience." "I'd lie to have been able to kick the fellow down the steps, siad mr. shelby to himself, when the trader had bowed himself himself, when the trader had bowed humself out. And Eliza's child. too I know I shal havesome fuss with the wife about that, and for that matter, about tom, too so much for being in dibt, heigho the prayer-meeting at uncle tome's cabin had been protracted to a very late hour, and Tom and his worthy helpmeet were not yet asleep, when between twelve and one there was a light tap on the window pane good Lord what that said Aunt chloe, starting up. My sakes alive it it aint Lizzy Get on your clothes, old man, quick. I'm gwine to open the door. and suiting the action to the word, the door flew open and the light of the candle wich Tom had hastily lighted. fell on the face of elizxa. I' m running away. Uncle tom and aunt chloe__ carrying off my child. Master sold him. " " Sold him? echoed both. holding up their hands in dismay. " Yes. sold him said eliza firmly. ' I crept into ther closet by mistre4ss's door to night, and I heard master tel missus that he had sold my harry and you , Uncle tom, both to a trader, and that the man was to take posses- sion to - day" Slowly, as the meaning of this speech came over Tom, he collapsed on his old chair, and sunk his head on his knees. 8. It can reasonably be inferred that mr. shelby places the highest value on which character?

Answer»

tom
eliza
eliza's son
CHLOE

Solution :As evidenced by lines 41-42 Mr. Shelby is clear and resolute that Eliza will not be sold, since, the agrees_ ALBEIT grudgingly _to the sale of Tom and Eliza's son, it can be inferred that he PLACES a higher value on Eliza. The passage does not PROVIDE evidence either way for his copinion of chloe.